Practice MCQ Questions on Indian Geography | Indian Geography Quiz Questions with Answers

Indian Geography is a major subject in the part of General Awareness section for competitive exams & government exams. Exams like IAS, State PSC, SSC, UPSC, Bank, etc. will ask plenty of questions from Indian Geography. So, practice MCQ Questions on Indian Geography from this page and test your level of preparation. In this guide, we have provided various Indian Geography Quiz Questions and Answers with a clear-cut explanation.

Indian Geography Multiple Choice Questions & Answers

1. Kalgoorlie and Coolgardie – places famous for gold mines are located in:
A. Australia
B. USA
C. England
D. South Africa

Answer

Answer: Option A
Explanation: Kalgoorlie-Boulder, known colloquially as just Kalgoorlie, is a city in the Goldfields-Esperance region of Western Australia, located 595 km (370 mi) east-northeast of Perth at the end of the Great Eastern Highway.


2. Tadoba national park known for sheltering tiger, panther and bear is located in:
A. Assam
B. Chandrapur (Maharashtra)
C. Karnataka
D. Tamil Nadu

Answer

Answer: Option B
Explanation : In the Chandrapur district of Maharashtra lies the Tadoba Andhari National park which shelters around 43 tigers till date. The tiger reserve is one of the fifty tiger reserves in India which also happens to be the largest and oldest national park in the state. The name of the national park was kept after the name of the God “Tadoba” who is worshipped by the local tribes, and Andhari is the name of the river which meanders between the forest.


3. In geography who propounded the theory of convectional current hypothesis?
A. Arthur Holmes
B. Carl Ritter
C. Arnaldo Faustini
D. Immanuel Kant

Answer

Answer: Option A
Explanation : Arthur Holmes postulated convection current theory in the year 1928–29. It is widely accepted driving mechanism for continental drift(moving apart) which lead to the foundation of modern plate tectonics. His main objective was to find the scientific explanation of origin of the continent and oceans. But it is also widely accepted concept of mountain building.


4. Apatanis are the tribes found in:

A. Himanchal Pradesh
B. Nagaland
C. Sikkim
D. Arunachal Pradesh

Answer

Answer: Option D
Explanation : The Apatani, or Tanw, also known by Apa and Apa Tani, are a tribal group of people living in the Ziro valley in the Lower Subansiri district of Arunachal Pradesh in India.


5. Genoa is leading seaport of:
A. Spain
B. Brazil
C. Italy
D. Canada

Answer

Answer: Option C
Explanation : The Port of Genoa is the major Italian seaport and it’s in competition with the ports of Marseille and Barcelona to be the biggest of the Mediterranean Sea. With a trade volume of 51.6 million tonnes, it is the busiest port of Italy by cargo tonnage.


6. The year ____ is called a Great Divide in the demographic history of India.
A. 1901
B. 1921
C. 1941
D. 1951

Answer

Answer: Option B
Explanation : The year 1921 is a “year of the great divide” in the demographic history of India when mortality started to decline leading to acceleration in the rate of population growth (the second stage).


7. Willandra Lakes Region a UNESCO Heritage site is located in:
A. The USA
B. Australia
C. The UK
D. Germany

Answer

Answer: Option B
Explanation : The Willandra Lakes Region is a world heritage site in the far west region of New South Wales, Australia.


8. The only private sector refinery set up by Reliance Petroleum Ltd. is located at
A. Guwahati
B. Jamnagar
C. Mumbai
D. Chennai

Answer

Answer: Option B
Explanation : The Jamnagar Refinery is a private sector crude oil refinery owned by Reliance Industries Limited in Jamnagar, Gujarat, India.


9. The only state in India that produces saffron is
A. Assam
B. Himachal Pradesh
C. Jammu and Kashmir
D. Meghalaya

Answer

Answer: Option C
Explanation : Jammu and Kashmir is the only state in India producing saffron.


10. The headquarters of International Astronomical Union (IAU) is located in:
A. Geneva
B. Mexico city
C. New york
D. Paris (France)

Answer

Answer: Option D
Explanation : International Astronomical Union (IAU), senior body governing international professional astronomical activities worldwide, with headquarters in Paris. It was established in 1919 as the first of a series of international unions for the advancement of specific branches of science.


11. With which country in the year 2010 did India propose to develop a solar city in India?

A. Japan
B. Russia
C. South Africa
D. USA

Answer

Answer: Option A
Explanation : Japan is the country with which India propose to develop a solar city in India.


12. The Barak River is a famous river of:
A. The Western Ghat
B. Eastern Ghat
C. Satpura hills
D. Manipur hills

Answer

Answer: Option D
Explanation : The principal tributaries of the Barak in India are the Jirl, the Dhaleshwari, the Singla, the Longai, the Sonai and the Katakhal. From its source in the Manipur Hills near Mao Songsang, the river is known as the Barak River.


13. The headquarters of North Western Railway is located in:
A. Bhopal
B. Gwalior
C. Uday
D. Jaipur

Answer

Answer: Option D
Explanation : North Western Railway has its zonal headquarters at Jaipur. Furthermore, the zone is divided into four divisions headquartered at Jaipur, Ajmer, Bikaner and Jodhpur.


14. Three important rivers of the Indian subcontinent have their sources near the Mansarover Lake in the Great Himalayas. These rivers are
A. Indus Jhelum and Sutlej
B. Brahmaputra Sutlej and Yamuna
C. Brahmaputra Indus and Sutlej
D. Jhelum Sutlej and Yamuna

Answer

Answer: Option C
Explanation : Three important rivers of the Indian subcontinent have their sources near the Mansarover Lake in the Great Himalayas. These rivers are Brahmaputra, Indus and Sutlej.


15. The zonal soil type of peninsular India belongs to
A. red soils
B. yellow soils
C. black soils
D. older alluvium

Answer

Answer: Option A
Explanation : The zonal soil type of peninsular India belongs to red soils.


16. The largest river of all the west flowing rivers of the peninsular India is:
A. Tapti
B. Kaveri
C. Krishna
D. Narmada

Answer

Answer: Option D
Explanation : Unlike east flowing major rivers of peninsular India, Narmada and Tapi (Tapti) are west flowing rivers running parallel to Vindhyas and Satpura Ranges and do not form deltas. Narmada is the third longest river of peninsular India with a length of 1312 kilometers.


17. Between the valley of Narmada river in the north and the Tapti in the south lies:
A. The Satpura Range
B. The Vindhya Range
C. The Aravali Range
D. None of Above

Answer

Answer: Option A
Explanation : The Narmada and the Tapti are major rivers that drain into the Arabian Sea. The Narmada originates in eastern Madhya Pradesh ( India) and flows west across the state, through a narrow valley between the Vindhya Range and spurs of the Satpura Range. It flows into the Gulf of Khambhat.


18. Scunthorpe and Frodingham are:
A. The main mining centres of Britain
B. Agriculture centres of Britain
C. industrial centres of Britain
D. None of these

Answer

Answer: Option A
Explanation : Frodingham was a hamlet in Lincolnshire which has grown into a suburb of Scunthorpe, North Lincolnshire. Although the village lay directly to the south of what is now Scunthorpe town centre, the name Frodingham is now often used to refer to the area directly to the north of the town centre. They are the main mining centres of Britain.


19. First Metro Railway was inaugurated in Kolkata:
A. 1982
B. 1987
C. 1984
D. 1990

Answer

Answer: Option C
Explanation : The underwater metro plan was thought of when the first metro service was inaugurated in Kolkata in October 24, 1984 by former Prime Minister Indira Gandhi.


20. India’s first Stock Exchange Bombay Stock Exchange was established in:
A. 1875
B. 1884
C. 1882
D. 1880

Answer

Answer: Option A
Explanation : The Bombay Stock Exchange, popularly known as BSE, is the oldest one in Asia. BSE was established in 1875 as The Native Share and Stock Brokers Association. The Tokyo Stock Exchange was established in 1878. The Bombay Stock Exchange, popularly known as BSE, is the oldest one in Asia.


21. Umiam Multipurpose Power Project is located in:
A. Himachal Pradesh
B. Meghalya
C. Sikkim
D. Nagaland

Answer

Answer: Option B
Explanation : Umiam Lake (commonly known as Barapani Lake) is a reservoir in the hills 15 km (9.3 mi) north of Shillong in the state of Meghalaya, India. It was created by damming the Umiam River in the early 1960s. Umiam Multipurpose Power Project is located in Meghalaya.


22. The world largest producer of natural gas is:
A. America
B. England
C. Russia
D. India

Answer

Answer: Option C
Explanation : Russia is the world’s leading natural gas exporter and leading natural gas producer, while also the second largest oil exporter and largest oil producer, though Russia interchanges the latter status with Saudi Arabia from time to time.


23. Xinhua is the Official News Agency of:
A. Japan
B. China
C. South Korea
D. North Korea

Answer

Answer: Option B
Explanation : Xinhua News Agency or New China News Agency is the official state-run press agency of the People’s Republic of China.


24. The index which refers to moisture deficit or surplus in an area is called:
A. Moisture Index
B. Humidity
C. Moisture Number
D. Dew Point

Answer

Answer: Option A
Explanation : The term used instead of ‘moisture budget’, e.g. by C. W. Thornthwaite (1955), and calculated from the aridity and humidity indices, as I m = 100 × (S − D)/PE, where I m is the moisture index, S is the water surplus, D is the water deficit, and PE is the potential evapotranspiration.


25. The study of plant communities in relation to their habitats of a given ecosystem is known as:

A. Seneology
B. Synecology
C. Ecology
D. Ecosystem

Answer

Answer: Option B
Explanation : Synecology is the study of group of organisms of different species which are associated together as a unit in form of a community. Also known as community ecology. Autecology helps us to understand the relationships between individual plants and environment.


26. The instrument which measures the intensity of solar radiation is called:
A. Barometer
B. Energymeter
C. Solarimeter
D. Speedometer

Answer

Answer: Option C
Explanation : The solarimeter measures routine global solar radiation on a plane or level surface. It has a thermocouple junction-sensing element. The sensing element is coated with a highly stable carbon based non organic coating, which delivers excellent spectral absorption and long-term stability characteristics. The sensing element is housed under two concentric fitting glass domes.


27. Any wind blowing down the slope of a mountain is called:
A. Anabatic Wind
B. Katabatic Wind
C. Down Falling Wind
D. Slope Wind

Answer

Answer: Option B
Explanation : A katabatic wind (named from the Greek word katabasis, meaning “descending”) is the technical name for a drainage wind, a wind that carries high-density air from a higher elevation down a slope under the force of gravity. Such winds are sometimes also called fall winds; the spelling catabatic winds also occurs. Katabatic winds can rush down elevated slopes at hurricane speeds, but most are not as intense as that, and many are of the order of 10 knots (18 km/h) or less.


28. The zone of low atmospheric pressure and ascending air located at or near the equator is called:
A. Inter-Tropical Convergence Zone (ITCZ)
B. Outer-Tropical Convergence Zone
C. Intera-Tropical Convergence Zone
D. None of Above

Answer

Answer: Option A
Explanation : The Inter Tropical Convergence Zone, or ITCZ, is a belt of low pressure which circles the Earth generally near the equator where the trade winds of the Northern and Southern Hemispheres come together. It is characterised by convective activity which generates often vigorous thunderstorms over large areas.


29. Indian state known as garden spice is:

A. Karnataka
B. West Bengal
C. Kerala
D. Sikkim

Answer

Answer: Option C
Explanation : Kerala is one of the beautiful state in India, it is known for its rich sources. Some of the spices are: Pepper, Vanilla, Cardamom and Clove.


30. Rajiv Gandhi Environment Award is given for outstanding contribution to:
A. Health
B. Tourism
C. Agriculture
D. Contribution in the development of Neat technology

Answer

Answer: Option D
Explanation : This award is given to industrial units that make a significant contribution towards the development of new, or the innovative modification of existing, technologies or adoption and use of clean technologies and practices that substantially reduce or prevent environmental pollution. The award consist of a cash prize of Rupees Two lakh, trophy and citation.


31. Sandstone, conglomerate, limestone, shalepotash, loess, geyserites, chalk, coal, chert, halite are the examples of:
A. Igneous rock
B. Metamorphic
C. Sedimentary Rocks
D. None of Above

Answer

Answer: Option C
Explanation : Sandstone, conglomerate, limestone, shalepotash, loess, geyserites, chalk, coal, chert, halite are the examples of Sedimentary Rocks.


32. The second most important agro based industry of India after textiles is:
A. Dairy Industry
B. Pulp Industry
C. Processed Food Industry
D. Sugar Industry

Answer

Answer: Option D
Explanation : India is the second largest sugar producing country of the world after Brazil. In 1840, the first sugar industry was set up in Betia (Bihar). Sugar production is known in India since ancient times, but modern sugar industry in India developed in first decade of the twentieth century.


33. As per 2011 Census of India the state which registered lowest sex ratio (female per thousand males) is:
A. Rajsthan
B. Haryana
C. Odissa
D. Nagaland

Answer

Answer: Option B
Explanation : Sex ratio is used to describe the number of females per 1000 of males. In the Population Census of 2011 it was revealed that the population ratio in India 2011 is 943 females per 1000 of males. Haryana has the lowest sex ratio (877) in Indian states while in union territory Daman & Diu has lowest sex ratio of 618.


34. Selves are dense tropical rain forest of:
A. South America
B. North America
C. South Africa
D. Australia

Answer

Answer: Option A
Explanation : A dense tropical rain forest usually having a cloud cover, especially one in the Amazon basin of South America.


35. Seasonal movement of people and their herds from valley to mountain and vice-versa is known as:
A. Transgoing
B. Transhumance
C. Transcoming
D. Transmountain

Answer

Answer: Option B
Explanation : Transhumance is a type of pastoralism or nomadism, a seasonal movement of livestock between fixed summer and winter pastures. In montane regions (vertical transhumance), it implies movement between higher pastures in summer and lower valleys in winter. Herders have a permanent home, typically in valleys. Generally only the herds travel, with a certain number of people necessary to tend them, while the main population stays at the base. In contrast, horizontal transhumance is more susceptible to being disrupted by climatic, economic, or political change.


36. The industrial region of India which has the predominance of metal industries is:
A. Penisular Region
B. Chotanagpur Region
C. Delta Region
D. Brahmputra Region

Answer

Answer: Option B
Explanation : This region is located on the Chotanagpur plateau and extends over Jharkhand, Northern Orissa and western part of West Bengal. The birth and growth of this region is linked with the discovery of coal in Damodar Valley and iron ore in the Jharkhand-Orissa mineral belt. As both are found in close proximity, the region is known as the ‘Ruhr of India’.


37. The mean thickness of oceanic crust and continental crust is around:
A. 5 kilometer and 15 kilometer respectively.
B. 10 kilometer and 30 kilometer respectively.
C. 5 kilometer and 30 kilometer respectively.
D. 18 kilometer and 40 kilometer respectively.

Answer

Answer: Option C
Explanation : The mean thickness of oceanic crust and continental crust is around 5 kilometer and 30 kilometer respectively.


38. Mahe and Yanam are the parts of the Union Territory of:
A. Dadar Nagar Haveli
B. Chandigarh
C. Daman and Diu
D. Puducherry

Answer

Answer: Option D
Explanation : Puducherry lies in the southern part of the Indian Peninsula. The areas of Puducherry district and Karaikal district are bound by the state of Tamil Nadu, while Yanam district and Mahé district are enclosed by the states of Andhra Pradesh and Kerala respectively.


39. The most important breed of wool producing sheep in the world is:

A. Texel
B. Merino
C. Deper
D. Dorset

Answer

Answer: Option B
Explanation : By 1810 Australia boasted over 30,000 sheep and was one of the world Merino wool trade centers, together with the United States and Germany and by 1840 it was the most important Merino sheep grower, together with South Africa and New Zealand. The rest of the world focused on cross-bred and coarser-wool sheep breeds.


40. Saar is a famous field in:
A. The Netherlands
B. Australia
C. USA
D. Germany

Answer

Answer: Option D
Explanation : The Territory of the Saar Basin (German: Saarbeckengebiet, Saarterritorium; French: Le Territoire du Bassin de la Sarre) was a region of Germany occupied and governed by the United Kingdom and France from 1920 to 1935 under a League of Nations mandate.


41. The place/city located at the confluence of river Euphrates and Tigris is:
A. Sydeny
B. London
C. Doha
D. Baghdad

Answer

Answer: Option D
Explanation : The place/city located at the confluence of river Euphrates and Tigris is Baghdad. Major canals that flowed from the Euphrates into the Tigris in the vicinity of Baghdad.


42. Which of the rock formations in India is the richest in fossil content?
A. Rift Valley
B. Delta
C. Gondwana
D. None of Above

Answer

Answer: Option C
Explanation : The Marine Gondwana Fossil Park is a unique exposure of fossiliferous marine Permian rocks of the Talchir formation dating back to around 280-240 million years ago. Located in the Manendragarh area of Chattisgarh’s Koriya district, the fossil faunal assemblage is indicative of the transgression of sea in this part of India about 20 crores of years ago.


43. India’s first marine national park is located in:

A. The Gulf of Mannar
B. Bay of Bengal
C. Arabian Sea
D. The Gulf of Khambhat

Answer

Answer: Option A
Explanation : The Gulf of Mannar Marine National Park is a protected area of India consisting of 21 small islands (islets) and adjacent coral reefs in the Gulf of Mannar in the Indian Ocean. It lies 1 to 10 km away from the east coast of Tamil Nadu, India for 160 km between Thoothukudi (Tuticorin) and Dhanushkodi.


44. Singareni Coal fields is located in :
A. Narmada Valley
B. Karishna Valley
C. Godavari Valley
D. Kaveri Valley

Answer

Answer: Option C
Explanation : Singareni opencast coal mines at Manuguru on Godavari Valley Coalfields is located in the districts of Adilabad, Karimnagar, Khammam and Warangal in the Indian state of Telangana. It is the only coalfield in South India. It lies in the basin of the Godavari River.


45. Pochampat irrigation project lies on:
A. Narmada
B. Godavari
C. Krishna
D. Kaveri

Answer

Answer: Option B
Explanation : The Sri Rama Sagar Project also known as the Pochampadu Project is an Indian flood-flow project on the Godavari.


46. Which planet is known as sister of earth?
A. Mars
B. Venus
C. Mercury
D. Saturn

Answer

Answer: Option B
Explanation : Venus is very similar to Earth in size and mass – and so is sometimes referred to as Earth’s sister planet – but Venus has a quite different climate.


47. The study of universe is called:
A. Astrology
B. Astronomy
C. Cosmology
D. Universology

Answer

Answer: Option C
Explanation : Cosmology is the scientific study of the large scale properties of the universe as a whole. It is the study of universe as a whole: its birth, growth, shape, size and eventual fate.


48. The large natural ecosystem comprised of abiotic (land, air , water and soils of the concerned habitat) and biotic (plants, animals and micro-organisms) is called:
A. Genome
B. Abiome
C. Agenome
D. Biome

Answer

Answer: Option D
Explanation : The large natural ecosystem comprised of abiotic (land, air , water and soils of the concerned habitat) and biotic (plants, animals and micro-organisms) is called Biome.


49. Tilaiya Dam, Konar dam, Maithon Dam and Panchet Hill Dam have been constructed under:
A. The Narmada Valley Project
B. The Damodar Valley Project
C. The Krishna Valley Project
D. The Mahanadi Project

Answer

Answer: Option B
Explanation : Damodar Valley Corporation has a network of four major dams in Jharkhand, such as Maithon on Barakar river, Tilaiya on Barakar River, Konar Dam on Konar river,Panchet on Damodar river.


50. Titagarh, Amlai and Nepanagar are known for:
A. Sugar Industry
B. Ship Building
C. Iron and Steel Industry
D. Paper Industry

Answer

Answer: Option D
Explanation : Titagarh, Amlai and Nepanagar are known for Paper Industry.


51. The highest glacial lake of India located at the height of 17,745 feet in Garhwal Himalaya is:
A. Devtal
B. Roopkund
C. Kedartal
D. Homkund

Answer

Answer: Option A
Explanation : The highest glacial lake of India is Devtal which is located at the height of 17,745 feet AMSL in Garhwal Himalaya.


52. Project Tiger, one of the premier conservation efforts in the country was launched on:
A. 10 June 1975
B. 1 August 1973
C. 10 April 1975
D. April 1 1973

Answer

Answer: Option D
Explanation : Project Tiger is a tiger conservation programme launched in April 1, 1973 by the Government of India during Prime Minister Indira Gandhi’s tenure.


53. Tungbhadra Multipurpose Project is a joint undertaking of:
A. Andra Pradesh and Tamil Nadu
B. Tamil Nadu and Karnataka
C. Kerala and Karnataka
D. Andhra Pradesh and Karnataka

Answer

Answer: Option D
Explanation : It is a joint undertaking by the governments of Andhra Pradesh and Karnataka. The project comprises a dam across the Tungabhadra River near Mallapuram. It is a power project of Gujarat equipped with power generating sets manufactured by Bharat Heavy Electricals Limited was inaugurated on October 12,1977.


54. In 1990 India ratified the world Heritage Convention of 1977 identifying four natural sites of outstanding universal value namely:
A. Kaziranga National park (Assam)
B. Keoladeo National Park (Rajasthan)
C. Sundarbans Biosphere Reserve (West Bengal)
D. Nanda Devi Biosphere Reserve (Uttarakhand)
E. All of these

Answer

Answer: Option E
Explanation : In 1990 India ratified the world Heritage Convention of 1977 identifying four natural sites of outstanding universal value namely Kaziranga National park (Assam), Keoladeo National Park (Rajasthan), Sundarbans Biosphere Reserve (West Bengal) and Nanda Devi Biosphere Reserve (Uttarakhand).


55. At present biosphere reserves that are included in world Heritage:
A. Nilgiri and Nanda Devi
B. Nokrek Gulf of Mannar and Sunderban
C. Simlipol and Pachmarhi
D. Above all
E. None of these

Answer

Answer: Option D
Explanation : At present biosphere reserves that are included in world are Heritage Nilgiri, Nanda Devi, Nokrek, Gulf of Mannar, Sunderban, Simlipol and Pachmarhi.


56. The radiant energy received by the earth and its atmosphere from the sun is called:

A. Insolation
B. Isotherm
C. Insoenergy
D. Albedo

Answer

Answer: Option A
Explanation : The energy received by the earth is known as incoming solar radiation which in short is termed as insolation. As the earth is a geoid resembling a sphere, the sun’s rays fall obliquely at the top of the atmosphere and the earth intercepts a very small portion of the sun’s energy.


57. The layer of ocean water between the depth zone of 300 m- 1000 m characterized by sharp change of temperature in the vertical section of sea water is called:
A. Isoline
B. Insiline
C. Thermoarea
D. Thermocline

Answer

Answer: Option D
Explanation : Thermocline, oceanic water layer in which water temperature decreases rapidly with increasing depth. A widespread permanent thermocline exists beneath the relatively warm, well-mixed surface layer, from depths of about 200 m (660 feet) to about 1,000 m (3,000 feet), in which interval temperatures diminish steadily.


58. A group of populations of different species living in the same habitat and interacting with one another is called:
A. Ecological Community
B. Zoological Community
C. Biological Community
D. None of Above.

Answer

Answer: Option A
Explanation : Organisms live within an ecological community, which is defined as an assemblage of populations of at least two different species that interact directly and indirectly within a defined geographic area.


59. Sand stone, Conglomerate, Limestone, Shale, Chalk and Coal are example of:

A. Igneous Rocks
B. Sedimentary Rocks
C. Metamorphic Rocks
D. None of Above.

Answer

Answer: Option B
Explanation : Sedimentary rocks are formed by the accumulation of sediments. There are three basic types of sedimentary rocks. Clastic sedimentary rocks such as breccia, conglomerate, sandstone, siltstone, and shale are formed from mechanical weathering debris.


60. Dr. Waldimir Koppen – the great climatologist of Austria suggested in his new scheme of climatic classification:
A. Two major types of climate.
B. Four major types of climate.
C. Five major types of climate.
D. Three major types of climate.

Answer

Answer: Option C
Explanation : The Köppen climate classification scheme divides climates into five main climate groups: A (tropical), B (dry), C (temperate), D (continental), and E (polar).


61. The modern iron and steel industry in India had its beginning in 1870 when Bengal Iron Works Company was established in:
A. Kulti
B. Visnupur
C. Cossipore
D. None of Above

Answer

Answer: Option A
Explanation : Modern steel making in India began with the setting of first blast furnace of India at Kulti in 1870 and production began in 1874, which was set up by Bengal Iron Works.Tata Iron and Steel Company (TISCO) was established by Dorabji Tata in 1907, as part of his father’s conglomerate.


62. The planet which has maximum number of satellites is:
A. Saturn
B. Venus
C. Jupiter
D. Mars

Answer

Answer: Option C
Explanation : In 2019, Saturn has unseated Jupiter as the solar system’s most moon-bearing planet. Scientists discovered 20 previously unknown moons orbiting Saturn, which gives Saturn a grand total of 82 moons, flying past Jupiter’s 79.


63. Which river is known as Sorrow of Bihar?
A. Ganga River
B. Gandak River
C. Kosi River
D. Son River

Answer

Answer: Option C
Explanation : The Kosi River is known as the “Sorrow of Bihar” as the annual floods affect about 21,000 km2 (8,100 sq mi) of fertile agricultural lands thereby disturbing the rural economy.


64. The largest Indian mammal is:

A. Tiger
B. Hippopotamus
C. The Lion
D. Elephant

Answer

Answer: Option D
Explanation : The Asian (or Indian) elephant is the largest terrestrial mammal in Asia, with a maximum shoulder height of 3.2 m. It is smaller than the African elephant (Loxodonta africana), with relatively smaller ears, and the head (not the shoulder) is the highest part of the body.


65. The Indian Council of Forestry Research and Education (ICFRE) were created in:
A. 1985
B. 1987
C. 1989
D. 1990

Answer

Answer: Option B
Explanation : ICFRE is the largest organisation responsible for forestry research in India. ICFRE was created in 1987, under the Central Ministry of Environment and Forests (India), to direct and manage research and education in forestry sector in India. ICFRE is headed by a Director General with headquarters at Dehradun.


66. The inner planets include:

A. Mercury Venus Earth and Jupiter
B. Mercury Venus Saturn and Mars.
C. Saturn Jupiter Neptune and Uranus.
D. Mercury Venus Earth and Mars.

Answer

Answer: Option D
Explanation : The inner four planets are called terrestrial planets. That means that they are like the earth in some ways. The terrestrial planets are Mercury, Venus, Earth, Mars, and the earth’s moon have similar compositions and densities. These planets are close to the sun, rocky, and dense.


67. The mean thickness of oceanic crust is:
A. 10 kilometer.
B. 8 kilometer.
C. 5 kilometer.
D. 4 kilometer.

Answer

Answer: Option C
Explanation : The mean thickness of oceanic crust is 5 kilometer.


68. The first large scale modern machine tool factory set up in public sector is :
A. 1953
B. 1955
C. 1957
D. 1959

Answer

Answer: Option A
Explanation : The Hindustan Machine Tools (H.M.T.) is the first large scale modem machine tool factory set up in public sector at Bangalore in 1953, with Swiss collaboration.


69. The first shore-based steel plant in India is:
A. Durgapur
B. Salem
C. Bhadravati
D. Vishakhapatnam Steel Plant (VSP)

Answer

Answer: Option D
Explanation : Visakhapatnam Steel Plant was separated from SAIL and RINL was made the corporate entity of Visakhapatnam Steel Plant in April 1982. Vizag Steel Plant is the only Indian shore-based steel plant and is situated on 33,000 acres (13,000 ha), and is poised to expand to produce up to 20 MT in a single campus.


70. Golden Quadrilateral Comprising National Highways connects:
A. Delhi-Mumbai-Chennai-Hydrabad-Delhi
B. Delhi-Mumbai-Chennai-Kollata-Delhi
C. Delhi-Mumbai-Benglore-Kollata-Delhi
D. Delhi-Mumbai-Benglore-Kollata-Hydrabad

Answer

Answer: Option B
Explanation : Golden Quadrilateral Comprising National Highways connects Delhi-Mumbai-Chennai-Kollata-Delhi (by six lane wuper highways).


71. The diesel locomotive works engaged in producing diesel locos is located in:
A. Varanasi
B. Vizag
C. Durgapur
D. Tuticorin

Answer

Answer: Option A
Explanation : The Diesel Locomotive Works (DLW) in Varanasi, India, is a production unit owned by Indian Railways, that manufactures diesel-electric locomotives and its spare parts. It is the largest diesel-electric locomotive manufacturer in India. It is located on DLW to BHU road of the metropolitan city of Varanasi.


72. The Mountain system of Alps, Carpathians, Transylvanian, Balkans are located in:
A. South America
B. New Zealand
C. Europe
D. North America

Answer

Answer: Option C
Explanation : The Mountain system of Alps, Transylvanian, Balkans are located in Europe.


73. The Chambal Valley Project is a joint venture of:
A. Rajasthan and Gujrat
B. Madhya Pradesh and Gujrat
C. Rajasthan and Madhya Pradesh
D. Uttar Pradesh and Madhya Pradesh

Answer

Answer: Option C
Explanation : The Chambal Valley Project: This is a joint venture of Rajasthan and Madhya Pradesh initiated in 1954 on the Chambal River (the main tributary of the Yamuna). The project aims at harnessing the Chambal River for irrigation, power generation and for prevention and control of soil erosion in the valley.


74. The largest satellite in the solar system is:
A. Titan
B. Ganymede
C. Europa
D. Setebos

Answer

Answer: Option B
Explanation : Ganymede is by far the largest moon and orbits around planet Jupiter with a diameter of 5,262 kilometers.


75. Vedanthangal Bird Sanctuary is located in:
A. Karnataka
B. Assam
C. Kerala
D. Tamil Nadu

Answer

Answer: Option D
Explanation : Vedanthangal Bird Sanctuary is a 30-hectare protected area located in the Kancheepuram District and Madurantakam taluk of the state of Tamil Nadu, India.


76. The line at which a day is lost or gained is called:
A. International Gain line
B. International loss line
C. Line of Control day and night
D. International Date line

Answer

Answer: Option D
Explanation : The International Date Line (IDL) is an imaginary line of demarcation on the surface of Earth that runs from the North Pole to the South Pole and demarcates the change of one calendar day to the next. It passes through the middle of the Pacific Ocean, roughly following the 180° line of longitude but deviating to pass around some territories and island groups.


77. The study of moon is called:
A. Lunarlogy
B. Selenology
C. Moonology
D. None of Above

Answer

Answer: Option B
Explanation : In Greek, our moon is named “Selene,” as is the moon goddess of ancient Greek mythology. The English word “selenology,” or the study of the moon’s geology, derives from it.


78. The portion of incident radiation (energy) without heating that surface reflected back from a surface of a body is called:
A. Streto
B. Ironodo
C. Albedo
D. Thermedo

Answer

Answer: Option C
Explanation : The ratio between the total solar radiation falling (incident) upon a surface and the amount reflected without heating the earth, is called ALBEDO (expressed as a decimal or as a percentage). The earth’s average albedo is about 0.4 (40 percent) ; that is, 4/10 of the solar radiation is reflected back into space.


79. The largest port of India is:
A. Kandla Port
B. Okha Port
C. Mumbai Port
D. Kolkata Port

Answer

Answer: Option C
Explanation : Jawaharlal Nehru port is the largest container port in India and is also known by Nhava Sheva. It is situated in Maharashtra (Mumbai). It accounts for more than half of total container volumes handled at India’s 12 public ports and around 40 percent of the nation’s overall containerized ocean trade.


80. Dul Hasti Project and Salal project are located at:
A. River Chenab
B. River Chambal
C. River Damodar
D. River Godavari

Answer

Answer: Option A
Explanation : Dulhasti power station is run-of-the-river with pondage scheme with an installed capacity of 390 MW (3 X 130MW) to harness the hydropower potential of river Chenab. It is located in Kishtwar district of Jammu & Kashmir.


81. The Vernal equinox occurs on:
A. January 29
B. March 21
C. April 1
D. May 25

Answer

Answer: Option B
Explanation : The Vernal equinox occurs on March 21.


82. The autumnal equinox occurs on:
A. August 23
B. September 23
C. October 23
D. November 23

Answer

Answer: Option B
Explanation : The autumnal equinox occurs on September 23.


83. Indira Gandhi Zoological Park is located in:
A. Hyderabad
B. Kolkata
C. Vishakhapatnam
D. Ahmedabad

Answer

Answer: Option C
Explanation : Indira Gandhi Zoological Park is located amidst Kambalakonda Reserve Forest in Visakhapatnam, Andhra Pradesh, India. It is the third largest zoo in the country. The zoological park is named after the former Prime Minister of India, Indira Gandhi. It was declared open to the public on 19 May 1977.


84. The predominant gases that constitute the sun are:
A. Hydrogen and Helium
B. Hydrogen and Methane
C. Methane and Helium
D. Methane and CO2

Answer

Answer: Option A
Explanation : The predominant element in the Sun is hydrogen, and then helium: by mass, it is 70% hydrogen, 28% helium.


85. Madumalai Sanctuary is located in:
A. Karnataka
B. Tamil Nadu
C. Kerala
D. Maharashtra

Answer

Answer: Option B
Explanation : The Mudumalai National Park and Wildlife Sanctuary also a declared tiger reserve, lies on the northwestern side of the Nilgiri Hills, in Nilgiri District, about 150 kilometres north-west of Coimbatore city in Tamil Nadu. It shares its boundaries with the states of Karnataka and Kerala.


86. Exfoliation is a type of:
A. Chemical Weathering
B. Soil Erosion
C. Physical Weathering
D. None of Above

Answer

Answer: Option C
Explanation : Exfoliation occurs as cracks develop parallel to the land surface a consequence of the reduction in pressure during uplift and erosion. It is a type of physical weathering.


87. World Water Day is observed on:
A. 22 December
B. 22 March
C. 18 April
D. 10 May

Answer

Answer: Option B
Explanation : World Water Day is an annual UN observance day (always on 22 March) that highlights the importance of freshwater. The day is used to advocate for the sustainable management of freshwater resources.


88. The largest state of India(area wise ) is:
A. Maharastra
B. Madhya Pradesh
C. Uttar Pradesh
D. Rajasthan

Answer

Answer: Option D
Explanation : Rajasthan is a state in northern India. The state covers an area of 342,239 square kilometres (132,139 sq mi) or 10.4 percent of the total geographical area of India. It is the largest Indian state by area and the seventh largest by population.


89. Hyderabad (in Telangana) is located on the bank:
A. Luni River
B. Gandak River
C. Mahanadi
D. Musi River

Answer

Answer: Option D
Explanation : Situated in the southern part of Telangana in southeastern India, Hyderabad is 1,566 kilometres (973 mi) south of Delhi, 699 kilometres (434 mi) southeast of Mumbai, and 570 kilometres (350 mi) north of Bangalore by road. It lies on the banks of the Musi River, in the northern part of the Deccan Plateau.


90. The River Cauvery rises on the western edge of:
A. Aravali
B. The Western Ghats
C. Satpuda Hills
D. The Eastern Ghats

Answer

Answer: Option B
Explanation : Cauvery river rises at Talakaveri on the Brahmagiri range in the Western Ghats in Karnataka at an elevation of about 1341 m and flows for about 800 km before its outfall into the Bay of Bengal. The Cauvery river system consists of 21 principal tributaries each with catchment area exceeding 250 Sq.Kms.


91. Primary Producers fall in two categories e.g.
A. Indigotrops and Xylotrops
B. Numerotrops
C. Xylotrops
D. Phototrophs and Chemotrophs

Answer

Answer: Option D
Explanation : Phototrophs and chemotrophs are two types of nutritional groups found in the environment. Most phototrophs are autotrophs, using the energy from sunlight to produce their food. Chemotrophs oxidize inorganic compounds or organic compounds as their energy source. They are the primary producers of food chains.


92. The Naharkatiya oil field is located in:
A. Gujrat
B. Bihar
C. Assam
D. Madhya Pradesh

Answer

Answer: Option C
Explanation : Naharkatia (also spelled as Naharkatia or Nahorkatiya) is a town and a town area committee in Dibrugarh district in the Indian state of Assam. It is well known for petroleum and gas reserves. Earlier, Duliajan, the head office town of Oil India Limited was in its circle.


93. The largest mangrove forest in the world is located in:
A. Kerala
B. West Bengal
C. Tamil Nadu
D. Madhya Pradesh

Answer

Answer: Option B
Explanation : The Sundarbans is a mangrove area in the delta formed by the confluence of Ganges, Brahmaputra and Meghna Rivers in the Bay of Bengal. It spans from the Hooghly River in India’s state of West Bengal to the Baleswar River in Bangladesh.


94. The water body separating Andaman and Nicobar is known as:
A. 9 degree channel
B. 8 degree channel
C. 10 degree channel
D. Indira Point

Answer

Answer: Option C
Explanation : The Ten Degree Channel is a channel that separates the Andaman Islands and Nicobar Islands from each other in the Bay of Bengal. The two sets of islands together form the Indian Union Territory (UT) of Andaman and Nicobar Islands.


95. Sun’s rays can make right angle only up to 23(1/2) and South latitudes because:
A. Rotation
B. Revolution
C. Shape of Earth
D. Earth is inclined at an angle of 66(1/2) on its axis.

Answer

Answer: Option D
Explanation : Sun’s rays can make right angle only up to 23(1/2) and South latitudes because Earth is inclined at an angle of 66(1/2) on its axis.


96. Which industry is developed in Shahdol and Nepanagar?
A. Paper and Pulp Industry
B. Agro Industry
C. Agricultural Industry
D. Rubber Industry

Answer

Answer: Option A
Explanation : Paper and Pulp Industry is developed in Shahdol and Nepanagar.


97. The Richter scale on which intensity/magnitude of a quake is measured was developed in:
A. 1933
B. 1932
C. 1935
D. 1939

Answer

Answer: Option C
Explanation : Richter scale (ML), quantitative measure of an earthquake’s magnitude (size), devised in 1935 by American seismologists Charles F. Richter and Beno Gutenberg. The earthquake’s magnitude is determined using the logarithm of the amplitude (height) of the largest seismic wave calibrated to a scale by a seismograph.


98. Masai Tribes are found in:
A. Australia
B. North America
C. South Africa
D. East Africa

Answer

Answer: Option D
Explanation : Maasai are a Nilotic ethnic group inhabiting northern, central and southern Kenya and northern Tanzania. They are among the best known local populations internationally due to their residence near the many game parks of the East African Great Lakes, and their distinctive customs and dress.


99. The largest producers of coffee in the world is:
A. India
B. Brazil
C. China
D. USA

Answer

Answer: Option B
Explanation : Brazil is the largest coffee producing country in the world, and the history comes from last 150 years.


100. The law which states that wave-length of maximum radiation is inversely proportional to the absolute of the emitting body is known as:
A. Wien’s Displacement Law
B. Ohm’s Law
C. Ferel’s Law
D. Ampere’s Law

Answer

Answer: Option A
Explanation : The Wien’s Displacement Law states that a black body having a wavelength (λmax) carrying the maximum energy is inversely proportional to the absolute temperature (T).


101. The headquarters of North Eastern Railway is:

A. Lucknow
B. Patna
C. Kolkata
D. Gorakhpur

Answer

Answer: Option D
Explanation : The North Eastern Railway is one of the 18 railway zones in India. It is headquartered at Gorakhpur and comprises Lucknow, Varanasi and Izzatnagar or Bareilly division.


102. The Western Ghats are also known as:
A. The Sahyadris
B. Sivalik Range
C. Greater Mountain range
D. Satpuda Range

Answer

Answer: Option A
Explanation : The range is known as Sahyadri in Maharashtra and Karnataka. The Western Ghats meets the Eastern Ghats at Nilgiris in northwestern Tamil Nadu. Nilgiris connects Biligiriranga Hills in southeastern Karnataka with the Shevaroys and Tirumala hills.


103. Ankleshwar Basin (offshore) oil-fields is located in:
A. Gujarat
B. Assam
C. Madhya Pradesh
D. Maharashtra

Answer

Answer: Option A
Explanation : Ankleshwar Basin (offshore) oil-fields is located in Gujarat.


104. The major peaks like Mt. Everest, Kanchenjunga, Makalu, Dhaulagiri, Mansalu, Chooyu, Nanga Parbat and Annapurna all are situated in:
A. The Western Ghat
B. Sivalik Range
C. The Greater Himalayas (Himadri)
D. The Eastern Ghats

Answer

Answer: Option C
Explanation : The major peaks like Mt. Everest, Kanchenjunga, Makalu, Dhaulagiri, Mansalu, Chooyu, Nanga Parbat and Annapurna all are situated in The Greater Himalayas (Himadri).


105. The Highest peak of the Karakoram Range is:
A. Kanchanjhangha
B. Mt. K2 (Godvin Austin )
C. Dhawalagiri
D. Anay Mudi

Answer

Answer: Option B
Explanation : K2 also known as Mount Godwin-Austen or Chhogori at 8,611 metres (28,251 ft) above sea. K2 is the highest point of the Karakoram range and the highest point in both Pakistan and Xinjiang.


106. India is divided into:

A. Six Postal Zones.
B. Seven Postal Zones.
C. Eight Postal Zones.
D. Nine Postal Zones.

Answer

Answer: Option D
Explanation : There are nine PIN zones in India, including eight regional zones and one functional zone (for the Indian Army). The first digit of the PIN code indicates the region. The second digit indicates the sub-region, and the third digit indicates the sorting district within the region.


107. Rail coach factory is located in:

A. Durgapur
B. Vizag
C. Varanasi
D. Kapurthala (Punjab)

Answer

Answer: Option D
Explanation : Rail Coach Factory at Kapurthala in the Indian state of the Punjab is located on the Jalandhar-Firozpur line.


108. Vembanad Lake is located in:
A. Maharastra
B. Madhya Pradesh
C. Karnataka
D. Kerala

Answer

Answer: Option D
Explanation : Vembanad is the longest lake in India, and the largest lake in the state of Kerala. Spanning several districts in the state of Kerala, it is known as Vembanadu Lake in Kottayam, Punnamada Lake in Kuttanad and Kochi Lake in Kochi.


109. India’s first Atomic power plant is:
A. Kalpakkam
B. Tarapur
C. Narora
D. Cossipore

Answer

Answer: Option B
Explanation : Tarapur Atomic Power Station (T.AP.S.) was the first nuclear power plant in India. The construction of the plant was started in 1962 and the plant went operational in 1969. The 320 MW Tarapur nuclear power station housed two 160 MW boiling water reactors (BWRs), the first in Asia.


110. Where is the volcanic Mt. St. Helens situated?
A. United States of America
B. UK
C. Canada
D. Australia

Answer

Answer: Option A
Explanation : Mount St. Helens is an active stratovolcano located in Skamania County, Washington, in the Pacific Northwest region of the United States of America. It is 50 miles northeast of Portland, Oregon and 96 miles south of Seattle, Washington.


111. Temperate grasslands of Eurasia are called:
A. Grasswood
B. Getepp
C. Steppe
D. Greenwood

Answer

Answer: Option C
Explanation : The Eurasian Steppe, also called the Great Steppe or the steppes, is the vast steppe ecoregion of Eurasia in the temperate grasslands, savannas, and shrublands biome.


112. The circulation and movement of soluble inorganic matter (nutrients) derived from sedimentary and atmospheric phases and reservoirs through organic phase of various biotic components and finally their return to inorganic phase is collectively called:
A. Chemical Cycles
B. Biogeochemical Cycles
C. Physical Cycles
D. Biological Cycles

Answer

Answer: Option B
Explanation : The circulation and movement of soluble inorganic matter (nutrients) derived from sedimentary and atmospheric phases and reservoirs through organic phase of various biotic components and finally their return to inorganic phase is collectively called Biogeochemical Cycles.


113. The middle zone of the earth with a thickness of 2780 km having an average density of 5.6 is known as:
A. Rosphere
B. Osophere
C. Pyrosphere
D. Cyrosphere

Answer

Answer: Option C
Explanation : The middle zone of the earth with a thickness of 2780 Km having an average density of 5-6 is known as Pyrosphere.


114. The instrument which records the seismic waves generated by the occurrence of earth-quakes is called:
A. Barometer
B. Gyrometer
C. Wavograph
D. Seismograph

Answer

Answer: Option D
Explanation : Seismograph, instrument that makes a record of seismic waves caused by an earthquake, explosion, or other Earth-shaking phenomenon. A record produced by a seismograph on a display screen or paper printout is called a seismogram.


115. Wild Ass Sanctuary is situated in
A. Assam
B. Gujarat
C. Madhya Pradesh
D. Maharashtra

Answer

Answer: Option B
Explanation : Indian Wild Ass Sanctuary also known as the Wild Ass Wildlife Sanctuary is located in the Little Rann of Kutch in the Gujarat state of India. It is spread over an area of 4954 km² .


116. The most important coal field in India is:
A. Jharia Coalfield
B. Rani Ganj Coalfield
C. Korba Coalfield
D. Singrauli Coalfield

Answer

Answer: Option A
Explanation : Jharia Coalfield is a large coal field located in the east of India in Jharia, Jharkhand. Jharia represents the largest coal reserves in India having estimated reserves of 19.4 billion tonnes of coking coal.


117. World Environment Day is observed on:
A. March 5
B. April 5
C. June 5
D. July 5

Answer

Answer: Option C
Explanation : World Environment Day (WED) is celebrated on the 5th of June every year, and is the United Nation’s principal vehicle for encouraging awareness and action for the protection of our environment.


118. The Beas project consisting of Beas-Satluj link and pong dam at Beas is the joint venture of:
A. Punjab Gujarat and Rajasthan
B. Punjab Haryana and Gujarat
C. Punjab Haryana and Rajasthan
D. Gujarat Haryana and Rajasthan

Answer

Answer: Option C
Explanation : The Beas project consisting of Beas-Satluj link and pong dam at Beas is the joint venture of Punjab, Haryana and Rajasthan.


119. The crude Birth Rate is defined as:
A. No. of births per 10 thousand populations
B. No. of births per 20 thousand populations
C. No. of births per 50 thousand populations
D. No. of births per thousand populations

Answer

Answer: Option D
Explanation : The crude birth rate is the number of live births occurring among the population of a given geographical area during a given year, per 1,000 mid-year total population of the given geographical area during the same year.


120. McMohan Line lies between which of the following countries?

A. India and Pakistan
B. India and China
C. China and Pakistan
D. India and Afghanistan

Answer

Answer: Option B
Explanation : It is currently the effective boundary between China and India, although its legal status is disputed by the Chinese government. The line is named after Henry McMahon, foreign secretary of British India and the chief negotiator of the convention at Simla.


121. India is the seventh largest country in the world and occupies:
A. 7% of world area
B. 5% of world area
C. 4% of world area
D. 2.4% of world area

Answer

Answer: Option D
Explanation : India is the seventh largest country in the world, following Russia, Canada, USA, China, Brazil and Australia, in that order. It occupies 2.4% of the world’s total land but supports 16.7% of the population.


122. The Nagarjunsagar Dam project is located on:
A. Mahanadi
B. Krishna River
C. Godavari
D. Kaveri

Answer

Answer: Option B
Explanation : Nagarjuna Sagar Dam was built across the Krishna river at Nagarjuna Sagar where the river forms the boundary between Nalgonda District in Telangana and Guntur district in Andhra Pradesh states in India. The construction duration of the dam was between the years of 1955 and 1967.


123. River Damodar is a tributary of:
A. Rive Koshi
B. River Gandak
C. River Sutlej
D. River Hoogly

Answer

Answer: Option D
Explanation : Damodar River, river in northeastern India, rising with its many tributaries, notably the Bokaro and Konar, in the Chota Nagpur plateau of south-central Bihar state. It follows a generally eastward course for 368 miles (592 km) through West Bengal to join the Hugli (Hooghly) River southwest of Kolkata (Calcutta).


124. The Himalayas were uplifted from the:
A. Valcano
B. Plateau
C. Grass land
D. Tethys Geosynclines

Answer

Answer: Option D
Explanation : The excessive shrinking of the geosynclines, caused folding and the surplus sediments were raised as the Himalayan mountains. The Himalayan uplift out of the tethys sea and the subsidence of the northern flank of the peninsular plateau resulted in the formation of a large basin.


125. In India the percentage of population below poverty line is:
A. Less than 30%
B. Less than 50%
C. Less than 40%
D. Less than 10%

Answer

Answer: Option A
Explanation : India accounted for the largest number of people living below international poverty line in 2013, with 30 per cent of its population under the $1.90-a- day poverty measure, the World Bank said.


126. The imaginary line joining the land projecting towards the sea which is the landward limit of internal seawater is called:

A. Base Line
B. Sea Line
C. Land Line
D. None of Above

Answer

Answer: Option A
Explanation : The imaginary line joining the land projecting towards the sea which is the landward limit of internal seawater is called Base Line.


127. CGWB stands for:
A. Central Ground Water Board
B. Central Ground Water and irigation Board
C. Central Ground Welfare Board
D. None of Above

Answer

Answer: Option A
Explanation : Central Ground Water Board (CGWB), a subordinate office of the Ministry of Water Resources, Government of India, is the National Apex Agency entrusted with the responsibilities of providing scientific inputs for management, exploration, monitoring, assessment, augmentation and regulation of ground water resources of the country. Central Ground Water Board was established in 1970 by renaming the Exploratory Tube wells Organization under the Ministry of Agriculture, Government of India. It was merged with the Ground Water Wing of the Geological Survey of India during 1972.


128. Sundarban was declared a World Heritage Site because of its:
A. River Delta
B. Mangroves Forests and Biodiversity
C. Nearest to Bay of Bengal
D. None of Above

Answer

Answer: Option B
Explanation : The Sundarbans mangrove forest, one of the largest such forests in the world (140,000 ha), lies on the delta of the Ganges, Brahmaputra and Meghna rivers on the Bay of Bengal. It is adjacent to the border of India’s Sundarbans World Heritage site inscribed in 1987.


129. The Greenhouse effect gases trap only the heat of:
A. All heat
B. UV Rays
C. Solar Radiation
D. Thermal Radiation

Answer

Answer: Option C
Explanation : The Greenhouse effect gases trap only the heat of Solar Radiation.


130. Which planet is called watery planet?
A. Venus
B. Earth
C. Mars
D. Mercury

Answer

Answer: Option B
Explanation : If you look down at our planet from outer space, most of what you see is water; 71% of the planet’s surface is covered by ocean and it is because of this that the Earth is sometimes called “the water planet”. Only about three-tenths of our globe is covered with land.


131. India shares maximum length of the border with:
A. Pakistan
B. Myanmar
C. Bangladesh
D. China

Answer

Answer: Option C
Explanation : India share its longest border with Bangladesh and that is 4093 km. Total length of the boundary, India shares with other countries is 15200 km.


132. Where is Silicon Valley located in?

A. U. S. A
B. UK
C. Green Land
D. Russia

Answer

Answer: Option A
Explanation : Silicon Valley, in the southern San Francisco Bay Area of California, U. S. A. is home to many start-up and global technology companies. Apple, Facebook and Google are among the most prominent. It’s also the site of technology-focused institutions centered around Palo Alto’s Stanford University.


133. The highest percentage decadal growth rate of population as per 2011 Census was registered by:
A. Maharastra
B. Punjab
C. Haryana
D. Meghalaya

Answer

Answer: Option D
Explanation : The highest percentage decadal growth rate of population as per 2011 Census was registered by Meghalaya (27.8).


134. Ken, Betwa and Chambal are the rivers which merge with:
A. Damodar
B. Yamuna
C. Ganga
D. Godavari

Answer

Answer: Option B
Explanation : Chambal and Betwa join Yamuna at the Etawah district and Hamirpur town in Uttarpradesh. As we move to the East, Yamuna Joins Ganga at Triveni Sangam, Allahabad.Son River joins River Ganga near Patna, Bihar.


135. Cotton in India grows best on:
A. Black Soil
B. Red Soil
C. Late-rite Soil
D. Sandy Soil

Answer

Answer: Option A
Explanation : Black soils are poor in nitrogen, phosphorus and organic matter. The soils are generally rich in the montmorillonitic and beidellitic group of clay minerals. It is most suitable for the growth of cotton. It is also known or cotton soil as a reason.


136. What is the difference between the Indian Standard time and Greenwich Standard Time?
A. 5 Hour
B. 15 Hour and 30 minutes
C. 5 Hour and 30 minutes
D. 6 Hour and 30 minutes

Answer

Answer: Option C
Explanation : The difference between GMT and IST is it varies in time duration that is IST is 5 hours and 30 minutes ahead of GMT.


137. Project Tiger in India was started in:
A. The year 1985
B. The year 1982
C. The year 1980
D. The year 1973

Answer

Answer: Option D
Explanation : One of the most intensive conservation efforts in India, Project Tiger was launched in 1973. It envisaged the setting up of tiger reserves to help in maintaining the population of tigers. As per estimates, the number of tigers was about 40,000 at the beginning of the 20th century.


138. Lignite is a type of:
A. Iron
B. Steel
C. Coal
D. Gold

Answer

Answer: Option C
Explanation : Lignite, often referred to as brown coal, is a soft, brown, combustible, sedimentary rock formed from naturally compressed peat. It is considered the lowest rank of coal due to its relatively low heat content. It has a carbon content around 60–70 percent.


139. One horned rhinoceros is found in the states of:
A. West Bengal and Madhya Pradesh
B. West Bengal and Assam
C. Madhya Pradesh and Assam
D. West Bengal and Uttar Pradesh

Answer

Answer: Option B
Explanation : The preferred habitat of an Indian rhinoceros is alluvial flood plains and areas containing tall grasslands along the foothills of the Himalayas. Formerly, extensively distributed in the Gangetic plains, today the species is restricted to small habitats in Indo-Nepal terai and northern West Bengal and Assam.


140. Wainganga river is a tributary of:
A. Kaveri
B. Godavari
C. Mahanadi
D. Gomati

Answer

Answer: Option B
Explanation : Wainganga (IAST: Wainagaṅgā) is a river in India, originating in the Mahadeo Hills in Mundara near village Gopalganj in Seoni Madhya Pradesh. It is a tributary of the Godavari River.


141. A river drains the water collected from a specific area which is called its:
A. Catchment Area
B. Collecting Area
C. Drain Area
D. None of Above

Answer

Answer: Option A
Explanation : A river drains the water collected from a specific area, which is called its ‘catchment area’. An area drained by a river and its tributaries is called a drainage basin. The boundary line separating one drainage basin from the other is known as the watershed.


142. The low flat central area of a basin of inland drainage is called:
A. Playa
B. Affusion
C. Afet
D. Aquifer

Answer

Answer: Option A
Explanation : Playas occupy the flat central basins of desert plains. They require interior drainage to a zone where evaporation greatly exceeds inflow. A saline playa may be called a salt flat, salt marsh, salada, salar, salt pan, alkali flat, or salina.


143. Which plateau lies between the Aravalli and the Vindhya ranges?
A. Malwa Plateau
B. Deccan Plateau
C. Chota Nagpur Plateau
D. None of Above.

Answer

Answer: Option A
Explanation : In the north-west are the Aravali range, running in south-west, north-east direction forming a discontinuous ridge. Thar Desert lies to the west of Aravali ranges. The southern boundary is demarcated by the Vindhya Range with Kaimur Hills in the eastern extent. The Malwa plateau lies between Aravalis and Vindhyas.


144. Green Index has been developed by:
A. WWF
B. UNESCO
C. United Nations Environment Programme
D. None of Above

Answer

Answer: Option C
Explanation : Green Index has been developed by United Nations Environment Programme.


145. The only South American nation through which both the Equator and the Tropic of Capricorn pass through?
A. Brazil
B. Ecuador
C. Peru
D. Guyana

Answer

Answer: Option A
Explanation : The Tropic of Capricorn lies at 23.5 degrees south of the equator and runs through Australia, Chile, southern Brazil (Brazil is the only country that passes through both the equator and a tropic), and northern South Africa.


146. The second zone of the interior of the earth extending from 30 kilometer to 2900 kilometer depth is:
A. Earth Crust
B. Mantle.
C. Outer Core
D. Inner Core

Answer

Answer: Option B
Explanation : The upper mantle begins at a depth of from 5 to 50 kilometers (3−30 miles) and extends to a depth of approximately 670 kilometers (400 miles) from the surface; the lower mantle extends from a depth of about 670 kilometers (400 miles) to about 2,900 kilometers (1,740 miles).


147. The average density of the outer-most layer (zone)- that is crust- of the earth is:
A. 1.2 to 5.0
B. 2.4 to 5.0
C. 2.1 to 3.0
D. 2.8 to 3.0

Answer

Answer: Option D
Explanation : The average density of the upper crust is 2.8 and that of the lower crust is 3.0. This difference in density is believed to be due to the pressure.


148. India’s three fourths of the total annual rainfall are received during:
A. South-west Monsoon Season
B. North East Monsoon Season
C. The Hot Weather Season
D. The Retreating of S.W. Monsoon Season

Answer

Answer: Option A
Explanation : The Arabian Sea current of the south- west monsoon which brings rain to the west coast of India. Much of the rainfall along the Western Ghats is orographic as the moist air is obstructed and forced to rise along the Ghats. Three fourths of the total annual rainfall is received during this season. The average rainfall over the plains of India in this season is about 87 per cent.


149. Paulo Alfonso hydroelectric power plant is located in:
A. Germany
B. Portugal
C. Brazil
D. Russia

Answer

Answer: Option C
Explanation : The Paulo Afonso Hydroelectric Complex (Complexo Hidrelétrico de Paulo Afonso), also known as the Paulo Afonso Complex, is a system of three dams and five hydroelectric power plants on the São Francisco River near the city of Paulo Afonso in Bahia, Brazil.


150. The leading producer of wind-power in the world is:
A. China
B. Germany
C. Brazil
D. USA

Answer

Answer: Option A
Explanation : China has a installed capacity of 221 GW and is the leader in wind energy, with over a third of the world’s capacity. It has the world’s largest onshore wind farm with a capacity of 7,965 megawatt (MW), which is five times larger than its nearest rival.


151. The state of atmosphere in relation to its water vapour content refers to:
A. Relative Humidity
B. Relative Density
C. Precipitation
D. Frontal Humidity

Answer

Answer: Option A
Explanation : Relative Humidity is the amount of water vapour present in air. Water vapour, the gaseous state of water, is generally invisible to the human eye. Humidity indicates the likelihood for precipitation, dew, or fog to be present. The amount of water vapour needed to achieve saturation increases as the temperature increases.


152. Biospheric ecosystem consist of two major components e.g.
A. Biome and Habitat Component
B. Habitat Component and VertualComponent
C. Vertical Component and Biome Component
D. None of the above

Answer

Answer: Option A
Explanation : Biospheric ecosystem consist of two major components e.g. Biome Component (Plants and Animals) and Habitat Component (Physical Environment).


153. Solar energy received to earth takes:
A. Nine Minutes
B. 10 Minutes
C. Eight Minutes
D. 15 Minutes

Answer

Answer: Option C
Explanation : It takes sunlight an average of 8 minutes and 20 seconds to reach to the Earth.


154. Garampani Wildlife Sanctuary is located in:
A. West Bengal
B. Assam
C. Madhya Pradesh
D. Karnataka

Answer

Answer: Option B
Explanation : Garampani Wildlife Sanctuary is a 6.05-square-kilometre (2.34 sq mi) wildlife sanctuary located in Karbi Anglong district, Assam, India. It is 25 km (16 mi) from Golaghat.


155. Sabarmati, Mahi, Narmada and Tapti rivers form:
A. Delta
B. Potholes
C. Estuaries
D. None of Above

Answer

Answer: Option C
Explanation : Sabarmati, Mahi, Narmada and Tapti rivers form Estuaries.


156. The word “tsunami”belongs to:
A. Chinese
B. German
C. English
D. Japanese

Answer

Answer: Option D
Explanation : Tsunami is a Japanese word from a double root: tsu, meaning port or harbour, and nami, meaning wave. The word looks innocuous in simple translation, but to those who live on the rim of the Pacific it can spell disaster. Tsunamis are fast moving ocean waves which spread across the open water like ripples on a pond.


157. The national Thermal Power Corporation (NTPC) was set up in:
A. 1970
B. 1975
C. 1978
D. 1980

Answer

Answer: Option B
Explanation : The company was founded on 7 November 1975 as “National Thermal Power Corporation Private Limited”. In 1983, NTPC began commercial operations (of selling power) and earned profits of INR 4.5 crores in FY 1982-83. By the end of 1985, it had achieved power generation capacity of 2000 MW.


158. Dharan, Abquiq , Quatif, Ain Dar, Ghawar and Safanija are the chief oil fields (petroleum regions) located in:
A. Egypt
B. Iran
C. Iraq
D. Saudi Arabia

Answer

Answer: Option D
Explanation : Dharan, Abquiq , Quatif, Ain Dar, Ghawar and Safanija are the chief oil fields (petroleum regions) located in Saudi Arabia.


159. Pykara hydroelectric power project is located at:
A. Godavari
B. Kaveri
C. Pykara River
D. Mahanadi

Answer

Answer: Option C
Explanation : The project is located at a distance of 16km downhill of the Pykara (Glenmorgan) power station. The Moyar HS also benefits by the water that is diverted from the Maravakandy Dam (located north-east of Masinagudi) through a flume channel.


160. The oil-gas fields Dighboi, Naharkatiya and Moran are located in:
A. Assam
B. Gujarat
C. Maharastra
D. Madhya Predesh

Answer

Answer: Option A
Explanation : Naharkatia (also spelled as Naharkatia or Nahorkatiya) is a town and a town area committee in Dibrugarh district in the Indian state of Assam. It is well known for petroleum and gas reserves. Earlier, Duliajan, the head office town of Oil India Limited was in its circle. However, the towns are close, within a 30-minute journey.


161. Which is known as thunder cloud?
A. Mid-Level Clouds
B. High-Level Clouds
C. Low-level Clouds
D. Cumulonimbus cloud

Answer

Answer: Option D
Explanation : Cumulonimbus (from Latin cumulus, “heaped” and nimbus, “rainstorm”) is a dense, towering vertical cloud, forming from water vapour carried by powerful upward air currents. If observed during a storm, these clouds may be referred to as thunderheads.


162. Cement industry is generally known as:

A. Source of raw material
B. Real Industry
C. Source of end products
D. All of the above

Answer

Answer: Option A
Explanation : Cement industry is generally known as source of raw material.


163. The first tidal power plant in India is located in:
A. Okha
B. Kochi
C. Vizag
D. Gulf of Kutch

Answer

Answer: Option D
Explanation : The Gujarat government is all set to develop India’s first tidal energy plant. The state government has approved Rs 25 crore for setting up the 50 MW plant at the Gulf of Kutch. It will produce energy from the ocean tides.


164. The Zasker range, the Ladakh range, the Kailas and the Karakoram Range are the main ranges of:
A. The Western Ghat
B. The Aravali
C. The Himalayan system
D. The Eastern Ghat

Answer

Answer: Option C
Explanation : The Zaskar range, the Ladakh range, the Kailas and the Karakoram Range are the main ranges of The Himalayan system.


165. The Sonapani glacier is the longest glacier of:
A. Sivalik range
B. Pir Panjal Range of the Himalayas
C. The Western Ghat
D. The Eastern Ghat

Answer

Answer: Option B
Explanation : Rising sharply to an average elevation of more than 13,000 feet (4,000 metres), it separates the Jammu Hills to the south from the Vale of Kashmir, beyond which lie the Great Himalayas. The major passes through the range include the Pir Panjal (11,462 feet [3,494 metres]) and Banihal (8,985 feet [2,739 metres]).


166. In India, the highest rainfall occurs along:
A. The West Coast North-East and hills of Meghalaya.
B. Pir Panjal Range of the Himalayas
C. The Eastern ghat
D. The Aravali

Answer

Answer: Option A
Explanation : In India, the highest rainfall occurs along the West Coast (on the Western Ghats), North-East and hills of Meghalaya.


167. Monpa, Daffla, Abor, Mishmi , Nishi and Nagas are the prominent tribal communities found in:
A. Sikkim
B. Himanchal Pradesh
C. Madhya Pradesh
D. Arunachal Pradesh

Answer

Answer: Option D
Explanation : Monpa, Daffla, Abor, Mishmi, Nishi and Nagas are the prominent tribal communities found in Arunachal Himalayas area.


168. The Gneiss is a coarse grained:
A. Volcanic Rock
B. Igneous Rock
C. Metamorphic Rock
D. Sedimentary Rock

Answer

Answer: Option C
Explanation : Gneiss is a common and widely distributed type of metamorphic rock. Gneiss is formed by high temperature and high-pressure metamorphic processes acting on formations composed of igneous or sedimentary rocks. Orthogneiss is gneiss derived from igneous rock (such as granite).


169. Gomardha Wildlife Sanctuary is located in:
A. Guhati
B. Raigarh
C. Gunna
D. Jaamnagar

Answer

Answer: Option B
Explanation : Gomarda wildlife sanctuary stretched across 277.82 sq km is 52 kilometer away from Raigarh and is named after the village Gomarda situated inside the sanctuary.


170. As per 2011 census data lowest decadal growth rate (state) has been registered in :
A. Kerala
B. West Bengal
C. Andhra Pradesh
D. Maharashtra

Answer

Answer: Option A
Explanation : As per 2011 census data lowest decadal growth rate (state) has been registered in Kerala (4.86%).


171. The leading producer of tea which accounts for about 28% of total production in the world is:
A. Kenya
B. India
C. Srilanka
D. Turkey

Answer

Answer: Option B
Explanation : India is world’s second largest tea producer after China. In 2013, India’s estimated tea production was 900 million kilograms, which counts for around 23-24% of global tea production. The top five producer countries of tea are China, India, Kenya, Sri Lanka and Turkey.


172. The narrow meandering bands of swift winds which blow in the midlatiludes near the tropopause and encircle the globe are known as:
A. Jet Stream.
B. Meander
C. Narrow wind band
D. None of these

Answer

Answer: Option A
Explanation : The narrow meandering bands of swift winds which blow in the mid latitudes near the tropo pause and encircle the globe are known as Jet Stream.


173. The first of the fourteen biosphere reserves of India which was established in 1986 is:
A. Sundarbans Biosphere Reserve
B. Gulf of Mannar Biosphere Reserve
C. Nilgiri Biosphere Reserve
D. Nanda Devi Biosphere Reserve

Answer

Answer: Option C
Explanation : The Nilgiri Biosphere Reserve was the first biosphere reserve in India established in the year 1986. It is located in the Western Ghats and includes 2 of the 10 biogeographical provinces of India. Wide ranges of ecosystems and species diversity are found in this region.


174. Nanda Devi Biosphere Reserve is located in:
A. West Bengal
B. Tamil Nadu
C. Meghalaya
D. Uttarakhand

Answer

Answer: Option D
Explanation : The Nanda Devi National Park or Nanda Devi Biosphere Reserve, established in 1982 , is a national park situated around the peak of Nanda Devi in the state of Uttarakhand in northern India. The entire park lies at an elevation of more than 3,500 m above mean sea level.


175. The average rainfall in India is about:
A. 100 cm
B. 125 cm
C. 110 cm
D. 140 cm

Answer

Answer: Option B
Explanation : The average rainfall in India is about 125 cm.


176. The highest peak of peninsular India is:
A. Dodabeta
B. Anaimudi
C. K2
D. Kanchanjangha

Answer

Answer: Option B
Explanation : Anamudi is the highest peak in the Western Ghats in India, having an elevation of 2,695 metres (8,842 ft). Anaimudi is also the highest point in South India.


177. Quartzite is metamorphosed from:
A. Limestone
B. Shale
C. Coal
D. Chalk

Answer

Answer: Option B
Explanation : Quartzite is a hard, non-foliated metamorphic rock which was originally pure quartz sandstone or Shale.


178. Limestone, Coal, Chalk, Shale are the examples of:
A. Sedimentary Rocks
B. Metamorphic Rock
C. Volcanic Rock
D. Igneous Rock

Answer

Answer: Option A
Explanation : Shale, sandstone, and limestone are the most common types of sedimentary rocks.


179. Granite and Basalt are the examples of:
A. Sedimentary Rocks
B. Metamorphic Rock
C. Volcanic Rock
D. Igneous Rocks

Answer

Answer: Option D
Explanation : Basalt and granite actually have quite a bit in common. Both are igneous rocks, which means that they cooled from a magma (the earth gets very hot just below the surface, and there is lots of liquid rock available). Both are made up of minerals from the silicate group, so both have large amounts of silicon and oxygen.


180. The Regur soil is:
A. Red Soil
B. Clay Soil
C. Black Soil
D. Sand Soil

Answer

Answer: Option C
Explanation : Black soil is also called Regur soil. It is black in colour and ideal for growing cotton. This type of soil is typical of the Deccan trap (Basalt) region spread over North-West Deccan plateau and is made up of lava flows.


181. The radius of earth is:
A. 6500 km
B. 6370 km
C. 6224 Km
D. 5553 Km

Answer

Answer: Option B
Explanation : The radius of the earth is 6370 km.


182. The crust and the upper-most part of the mantle up to a depth of 200 km from the surface of the earth is called:
A. Pyrosphere
B. Osophere
C. Cyrosphere
D. Lithosphere

Answer

Answer: Option D
Explanation : The mantle extends from Moho’s discontinuity (35 km) to a depth of 2,900 km (Moho-Discontinuity to the outer core). The crust and the uppermost part of the mantle are called lithosphere. Its thickness ranges from 10-200 km. The lower mantle extends beyond the asthenosphere.


183. The Earth’s core is made up of very heavy material mostly constituted by:
A. Iron and Aluminum
B. Aluminum and Nickel
C. Nickel and Iron
D. Nickel and Lead

Answer

Answer: Option C
Explanation : The solid, inner core of iron has a radius of about 760 miles (about 1,220 km), according to NASA. It is surrounded by a liquid, outer core composed of a nickel-iron alloy.


184. Earthquakes waves are of two types namely:
A. Body waves and surface waves
B. Circular and Body Waves
C. Body Waves and Vertical Waves
D. Circular and Vertical Waves

Answer

Answer: Option A
Explanation : The two main types of waves are body waves and surface waves. Body waves can travel through the earth’s inner layers, but surface waves can only move along the surface of the planet like ripples on water. Earthquakes radiate seismic energy as both body and surface waves.


185. The scale on which the intensity of an earth quake is measured is named after:

A. Newton
B. Einstein
C. Curie
D. Mercalli

Answer

Answer: Option D
Explanation : The Mercalli scale, named after Giuseppe Mercalli, measures the intensity of an earthquake by determining an earthquake’s effects.


186. The 99% mass of earth atmosphere is confines to the height of:
A. 16 Km from the Earth’s surface
B. 32 Km from the Earth’s surface
C. 64 Km from the Earth’s surface
D. 72 Km from the Earth’s surface

Answer

Answer: Option B
Explanation : Although our atmosphere extends upward for many hundreds of kilometers, 99% of the atmosphere’s mass is confined within a 30 km of the earth’s surface.


187. The gas which is largely responsible for green house effect is:
A. Ozone
B. Carbon dioxide
C. Methane
D. Water Vapor

Answer

Answer: Option D
Explanation : The greenhouse effect, in turn, is one of the leading causes of global warming. The most significant greenhouse gases are water vapor (H2O), carbon dioxide (CO2), methane (CH4) and nitrous oxide (N2O), according to the Environmental Protection Agency (EPA).


188. The column of atmosphere is divided into five different layers namely:
A. Troposphere Lithosphere Mesosphere Ionosphere Exosphere
B. Troposphere Stratosphere Mesosphere Pyrosphers Exosphere
C. Troposphere Stratosphere Lithosphere Ionosphere Pyrosphers
D. Troposphere Stratosphere Mesosphere Ionosphere Exosphere

Answer

Answer: Option D
Explanation : The column of atmosphere is divided into five different layers namely Troposphere, Stratosphere, Mesosphere, Ionosphere, Exosphere.


189. The most important layer for all biological activity in the atmosphere is:
A. Troposphere.
B. Stratosphere
C. Mesosphere
D. Ionosphere
E. Exosphere

Answer

Answer: Option A
Explanation : Troposphere plays important role in moderating the temperature on earth that makes it habitable. This delicate exchange of energy between the earth’s surface and atmosphere forbids the earth from becoming too hot and too cold.


190. The flood plain along river banks formed by newer alluvium is called:
A. Meander
B. River Valley
C. Delta
D. Khaddar

Answer

Answer: Option D
Explanation : Alluvium, material deposited by rivers. It is usually most extensively developed in the lower part of the course of a river, forming floodplains and deltas, but may be deposited at any point where the river overflows its banks or where the velocity of a river is checked—for example, where it runs into a lake. The flood plain along river banks formed by newer alluvium is called Khaddar.


191. The Paithan (Jayakwadi) Hydro-electric project, completed with the help of Japan, is on the river
A. Ganga
B. Cauvery
C. Narmada
D. Godavari

Answer

Answer: Option D
Explanation : The Paithan (Jayakwadi) Hydro-electric project, completed with the help of Japan, is on the river Godavari.


192. The percentage of irrigated land in India is about
A. 45
B. 65
C. 35
D. 25

Answer

Answer: Option C
Explanation : The total arable land in India is 160 million hectares (395 million acres). According to the World Bank, only about 35% of total agricultural land in India was reliably irrigated in 2010.


193. The southernmost point of peninsular India, that is, Kanyakumari, is
A. north of Tropic of Cancer
B. south of the Equator
C. south of the Capricorn
D. north of the Equator

Answer

Answer: Option D
Explanation : The southernmost point of peninsular India, that is, Kanyakumari, is north of the Equator.


194. The pass located at the southern end of the Nilgiri Hills in south India is called
A. the Palghat gap
B. the Bhorghat pass
C. the Thalgat pass
D. the Bolan pass

Answer

Answer: Option A
Explanation : The pass located at the southern end of the Nilgiri Hills in south India is called the Palghat gap.


195. Which of the following factors are responsible for the rapid growth of sugar production in south India as compared to north India?,

A. I and II
B. I II and III
C. I III and IV
D. I II and IV

Answer

Answer: Option D
Explanation : Factors are responsible for the rapid growth of sugar production in south India as compared to north India are Higher per acre field of sugarcane, Higher sucrose content of sugarcane and Longer crushing period.


196. Which city is known as the Cotton Polis of India?

A. Ahmedabad
B. Mumbai
C. Kochi
D. Kolkta

Answer

Answer: Option B
Explanation : Mumbai is called the cotton polis because it was the home of textile mills in England. Mumbai due to its high humidity was ideal for setting up cotton textile mills and at one time there were 130 textile mills in Bombay.


197. The largest producer of diamond In India is:
A. Madhya Pradesh
B. Gujarat
C. Maharastra
D. West Bengal

Answer

Answer: Option A
Explanation : Madhya Pradesh is the only diamond producing state and is leading producer of copper conc., pyrophyllite and diaspore. State hosts country’s 68% diaspore, 41% molybdenum ore, 46% pyrophyllite, 32% diamond, 29% copper ore, 17% rock phosphate, 16% each of manganese ore and fireclay and 11% ochre resources.


198. World forestry Day is observed on:
A. April 20 Every Year.
B. May 10 Every Year.
C. June 5 Every Year.
D. March 21 Every Year.

Answer

Answer: Option D
Explanation : World Forestry Day is celebrated annually on 21st March to raise awareness about the importance and conservation of forests all over the world.


199. India’s biggest super Thermal power station is located In:
A. Narora
B. Cossipore (Kolkata)
C. Koradi near Nagpur
D. Kalpakkam

Answer

Answer: Option C
Explanation : Koradi Thermal Power Station (KTPS) is located at Koradi near Nagpur, Maharashtra. The power plant is one of the four major power plants in Vidarbha – a power surplus region of India.


200. India established Durgapur, Rourkela and Bhilai steel plants in collaboration with:
A. Germany Britain and Russia respectively
B. Britain Germany and Russia respectively
C. Russia Germany and Britain respectively
D. Britain Russia and Germany respectively

Answer

Answer: Option B
Explanation : India established Durgapur, Rourkela and Bhilai steel plants in collaboration with Britain, Germany and Russia respectively. Durgapur – Britain, Rourkela – Germany, Bhilai – Russia.


201. The principal copper deposits of India lie in which of the following places?
A. Hazaribagh and Singbhum of Bihar
B. Khetri and Daribo areas of Rajasthan
C. Anantapur in Andhra Pradesh
D. Siwaliks in Uttar Pradesh and in Karnataka

Answer

Answer: Option A
Explanation : The principal copper belt of India lies in Singhbhum and Hazaribagh in Jharkhand. The mining centres are located at Mosabani, Ghatsila, Thobani and Badia of Singbhum, Hazaribagh of Bihar, the Khetri and Dariba areas of Rajasthan, and Agnigundala of Andhra Pradesh.


202. Which of the following are true regarding Jhum cultivation in India?,

A. I II and III
B. II and III
C. I and II
D. I and III

Answer

Answer: Option A
Explanation : The following that are true regarding Jhum cultivation in India are It is largely practiced in Assam, It is referred to as ‘slash and burn’ technique and In it, the fertility is exhausted in a few years.


203. The Yarlung Zangbo river, in India, is known as
A. Ganga
B. Indus
C. Brahmaputra
D. Mahanadi

Answer

Answer: Option C
Explanation : The Yarlung Zangbo river, in India, is known as Brahmaputra.


204. The Salal Project is on the river
A. Chenab
B. Jhelum
C. Ravi
D. Sutlej

Answer

Answer: Option A
Explanation : Salal Dam also known as Salal Hydroelectric Power Station, is a run-of-the-river power project on the Chenab River in the Reasi district of the Indian state of Jammu and Kashmir.


205. The only zone in the country that produces gold is also rich in iron is
A. North-eastern zone
B. North-western zone
C. Southern zone
D. None of the above

Answer

Answer: Option C
Explanation : The only zone in the country that produces gold is also rich in iron is Southern zone.


206. The percentage of earth surface covered by India is
A. 2.4
B. 3.4
C. 4.4
D. 5.4

Answer

Answer: Option A
Explanation : The percentage of earth surface covered by India is 2.4.


207. Which among the following is/are the major factor/factors responsible for the monsoon type of climate in India?
A. I
B. II III
C. II III and IV
D. I II III and IV

Answer

Answer: Option D
Explanation : The major factors responsible for the monsoon type of climate in India are location, thermal contrast, upper air circulation and inter-tropical convergence zone.


208. The present forest area of India, according to satellite data, is
A. increasing
B. decreasing
C. static
D. decreasing in open forest area but increasing in closed forest area

Answer

Answer: Option B
Explanation : The present forest area of India, according to satellite data is decreasing.


209. The India’s highest annual rainfall is reported at
A. Namchi Sikkim
B. Churu Rajasthan
C. Mawsynram Meghalaya
D. Chamba Himachal Pradesh

Answer

Answer: Option C
Explanation : Mawsynram is a village in the East Khasi Hills district of Meghalaya state in north eastern India, 65 kilometres from Shillong. Mawsynram receives one of the highest rainfalls in India.


210. The refineries Mathura, Digboi and Panipat are set up by
A. Indian Oil Corporation Ltd.
B. Hindustan Petroleum Corporation Ltd.
C. Bharat Petroleum Corporation Ltd.
D. Crude Distillation unit of Madras Refineries Ltd.

Answer

Answer: Option A
Explanation : The refineries are Mathura, Digboi and Panipat are set up by Indian Oil Corporation Ltd.


211. What is the predominant type of Indian agriculture?
A. Commercial agriculture
B. Extensive agriculture
C. Plantation agriculture
D. Subsistence agriculture

Answer

Answer: Option D
Explanation : The predominant type of Indian agriculture is subsistence agriculture.


212. The Radcliffe line is a boundary between
A. India and Pakistan
B. India and China
C. India and Myanmar
D. India and Afghanistan

Answer

Answer: Option A
Explanation : The Radcliffe Line, the border between the Union of India and the Dominion of Pakistan is revealed. On 17th August 1947 the Radcliffe Line was declared as the boundary between India and Pakistan, following the Partition of India.


213. Which of the following has a potential for harnessing of tidal energy in India?
A. Gulf of Cambay
B. Gulf of Mannar
C. Backwaters of Kerala
D. Chilka lake

Answer

Answer: Option A
Explanation : Gulf of Cambay has a potential for harnessing of tidal energy in India.


214. The typical area of sal forest in the Indian peninsular upland occurs
A. on the western ghats
B. between the Tapti and the Narmada
C. to the north-east of the Godavari
D. on the Malwa plateau

Answer

Answer: Option D
Explanation : The typical area of sal forest in the indian peninsular upland occurs on the Malwa plateau.


215. The state having a largest area of forest cover in India is
A. Arunachal Pradesh
B. Haryana
C. Madhya Pradesh
D. Assam

Answer

Answer: Option C
Explanation : Madhya Pradesh has the largest forest cover of 77,522 sq. km. in terms of area in the country followed by Arunachal Pradesh with the forest cover of 67,321 sq. km.


216. The most plausible explanation for the location of the Thar desert in western India is
A. the obstruction caused by the Aravalis to the rain-bearing wind that proceeds to the Ganga Valley
B. the evaporation of moisture by heat
C. the absence of mountains to the north of Rajasthan to cause orographic rainfall in it
D. that the moisture carried by the South-west monsoon is driven away by the dry upper air current

Answer

Answer: Option C
Explanation : The most plausible explanation for the location of the Thar desert in western India is the absence of mountains to the north of Rajasthan to cause orographic rainfall in it.


217. The northern boundary of the peninsular plateau of Indian runs parallel to the Ganga and the Yamuna from Rajmahal hills to a point near
A. Allahabad
B. Delhi
C. Gwalior
D. Jaipur

Answer

Answer: Option B
Explanation : The northern boundary of the peninsula plateau of India runs parallel to the Gange and the Yamuna from Rajmahal hills to a point near Delhi.


218. Which of the following food grain crops occupies the largest part of the cropped area in India?
A. Barley and maize
B. Jowar and bajra
C. Rice
D. Wheat

Answer

Answer: Option C
Explanation : Rice is the most cultivated crop in India. India is the world’s second-largest producer of rice, behind China. Rice was cultivated on an area of around 32.5 million hectares during the fiscal year 2002-2003.


219. The number of major languages, recognized in the Indian Union as official language, are
A. 15
B. 22
C. 12
D. 9

Answer

Answer: Option B
Explanation : The Eighth Schedule of the Indian Constitution lists 22 languages, which have been referred to as scheduled languages and given recognition, status and official encouragement. In addition, the Government of India has awarded the distinction of classical language to Kannada, Malayalam, Odia, Sanskrit, Tamil and Telugu.


220. The oldest rocks in India are reported from
A. Dharwar region Karnataka
B. Aravalli range Rajasthan
C. Vindhyan range Madhya Pradesh
D. Siwalik range Punjab

Answer

Answer: Option A
Explanation : Dharwad is the district headquarters of Dharwad district in the state of Karnataka, India. It was merged with the city of Hubballi in 1961 to form the twin cities of Hubballi-Dharwad. It covers an area of 200.23 km² and is located 425 km northwest of Bengaluru, on NH-48, between Bengaluru and Pune. The oldest rocks in India are reported from Dharwar region, Karnataka.


221. Which of the following groups of rivers originate from the Himachal mountains?
A. Beas Ravi and Chenab
B. Ravi Chenab and Jhelum
C. Sutlej Beas and Ravi
D. Sutlej Ravi and Jhelum

Answer

Answer: Option A
Explanation : The drainage systems of the region are the Chandra Bhaga or the Chenab, the Ravi, the Beas, the Sutlej and the Yamuna. These rivers are perennial and are fed by snow and rainfall. Groups of rivers originate from the Himachal mountains are Beas, Ravi and Chenab.


222. Which of the following groups of states has the largest deposits of iron ore?
A. Andhra Pradesh and Karnataka
B. Bihar and Orissa
C. Madhya Pradesh and Maharashtra
D. West Bengal and Assam

Answer

Answer: Option B
Explanation : Bihar and Orissa states have the largest deposits of iron ore.


223. Which of the following union territories of India has the highest density of population per sq km?
A. Pondicherry
B. Lakshadweep
C. Delhi
D. Chandigarh

Answer

Answer: Option C
Explanation : Delhi has a population density of 11,297 persons per sq km.


224. Which atomic power station in India is built completely indigenously?
A. Kalpakkam
B. Narora
C. Rawat Bhata
D. Tarapore

Answer

Answer: Option A
Explanation : Madras Atomic Power Station (MAPS) located at Kalpakkam about 80 kilometers (50 mi) south of Chennai, India, is a comprehensive nuclear power production, fuel reprocessing, and waste treatment facility that includes plutonium fuel fabrication for fast breeder reactors (FBRs) is the only atomic power station in India.


225. The south-west monsoon contributes ____ of the total rain in India.
A. 86%
B. 50%
C. 22%
D. 100%

Answer

Answer: Option A
Explanation : The south-west monsoon contributes around 86 percent of the total rain in India.


226. The Shimla Convention is an agreement that sets
A. Shimla as a tourist spot
B. Shimla as the capital of Himachal Pradesh
C. boundary between India and Tibet
D. None of the above

Answer

Answer: Option C
Explanation : India’s claim to a part of its north-east territories, for example, is largely based on the same agreements – notes exchanged during the Simla convention of 1914, which set the boundary between India and Tibet – that the British appear to have simply discarded.


227. Which of the following events took place in the Cenozoic era?
A. Formation of the rockies India collides with Asia and the formation of the Himalayas and the Alps
B. Formation of the Appalachians and central European mountains
C. Splitting of India from Antarctic
D. Breaking up of Pangaea

Answer

Answer: Option A
Explanation : Formation of the rockies, India collides with Asia and the formation of the Himalayas and the Alps took place in the Cenozoic era.


228. The oldest oil field in India is the ____ field, in ____
A. Anleshwar Gujarat
B. Bombay High Maharashtra
C. Nawagam Gujarat
D. Digboi Assam

Answer

Answer: Option D
Explanation : Digboi in Assam is an oil town that can be traced to the early 18th century, when oil was first discovered here. Digboi can proudly boast of two unique features: a 100-year-old extant oilfield and the world’s oldest operating oil refinery.


229. Unlike other parts of the Indian Coast, fishing industry has not developed along the Saurashtra coast because
A. there are few indentions suitable for fishing
B. of overwhelming dependence on agriculture and animal husbandary
C. the sea water is relatively more saline
D. of industrial development leading to widespread pollution of coastal area

Answer

Answer: Option B
Explanation : Unlike other parts of the Indian Coast, fishing industry has not developed along the Saurashtra coast because of overwhelming dependence on agriculture and animal husbandry.


230. The mountain building in Himalayas began
A. about 45 million years ago
B. when the continental plates of India and Eurasia converged on each other
C. both (a) and (b)
D. None of the above

Answer

Answer: Option C
Explanation : The mountain building in Himalayas began about 45 million years ago when the continental plates of India and Eurasia converged on each other.


231. The outer Himalayas lie between
A. the lease Himalayas and the Indo Gangetic plain
B. the foot hills and the Indo Gangetic plain
C. the greater Himalayas and the lesser Himalayas
D. Indo-Gangetic plains and the peninsula

Answer

Answer: Option A
Explanation : The outer Himalayas lie between the lease Himalayas and the Indo Gangetic plain.


232. Which of the following geographical features have played a great unifying role in strengthening the forces of homogeneity of the Indian people?,

A. I
B. II
C. I and II
D. I II III and IV

Answer

Answer: Option D
Explanation : Geographical features that have played a great unifying role in strengthening the forces of homogeneity of the Indian people are the expanses of water surrounding the peninsula, the Himalayan Mountains, the vastness of the country and the presence of the Indian ocean.


233. Which of the following drainage systems fall into Bay of Bengal?
A. Ganga Brahmaputra and Godavari
B. Mahanadi Krishna and Cauvery
C. Luni Narmada and Tapti
D. Both (a) and (b)

Answer

Answer: Option D
Explanation : Drainage systems fall into Bay of Bengal from Ganga, Brahmaputra, Godavari, Mahanadi, Krishna and Cauvery.


234. The oldest oil refinery in India is at
A. Digboi Assam
B. Haldia near Kolkata
C. Koyali near Baroda
D. Noonmati Assam

Answer

Answer: Option A
Explanation : Digboi in Assam is an oil town that can be traced to the early 18th century, when oil was first discovered here. Digboi can proudly boast of two unique features: a 100-year-old extant oilfield and the world’s oldest operating oil refinery.


235. The oldest mountains in India are
A. Aravalis
B. Vindhyas
C. Satpuras
D. Nilgiri hills

Answer

Answer: Option A
Explanation : The Aravalli Range, an eroded stub of ancient mountains, is the oldest range of fold mountains in India. The natural history of the Aravalli Range dates back to times when the Indian plate was separated from the Eurasian plate by an ocean.


236. Which of the following groups of rivers have their source of origin in Tibet?
A. Brahmaputra Ganges and Sutlej
B. Ganges Sutlej and Yamuna
C. Brahmaputra Indus and Sutlej
D. Chenab Ravi and Sutlej

Answer

Answer: Option C
Explanation : All the 10 major river systems of Asia including the Indus, Sutlej, Brahmaputra, Irrawady, Salween and Mekong originate in the Tibetan plateau. Among the following groups of rivers have their source of origin in Tibet are Brahmaputra, Indus and Sutlej.


237. Which of the following measures are effective for soil conservation in India?,

A. I and II
B. II and IV
C. III and IV
D. I II and III

Answer

Answer: Option B
Explanation : Measures that are effective for soil conservation in India are afforestation and Limiting shifting cultivation. Afforestation is the establishment of a forest or stand of trees (forestation) in an area where there was no previous tree cover. Shifting cultivation is an agricultural system in which plots of land are cultivated temporarily, then abandoned and allowed to revert to their natural vegetation while the cultivator moves on to another plot.


238. Which of the following crops needs maximum water per hectare?
A. Barley
B. Maize
C. Sugarcane
D. Wheat

Answer

Answer: Option C
Explanation : Sugarcane crops needs maximum water per hectare. The crop water need or crop evapotranspiration consists of transpiration by the plant and evaporation from the soil and plant surface.


239. The watershed between India and Myanmar is formed by
A. the Naga hills
B. the Garo hills
C. the Khasi hills
D. the Jaintia hills

Answer

Answer: Option A
Explanation : The water shed between India and Myanmar is formed by Naga hills. These lies in the borders of India and Burma.


240. The originating in the Himalayan mountain complex consists of how many distinct drainage systems of the Indian Subcontinent?
A. Two
B. Three
C. Four
D. Five

Answer

Answer: Option B
Explanation : The originating in the Himalayan mountain complex consists of three distinct drainage systems of the Indian subcontinent.


241. The percentage of India’s total population employed in agriculture is nearly
A. 40%
B. 50%
C. 60%
D. 70%

Answer

Answer: Option D
Explanation : The percentage of India’s population employed in agriculture is nearly 70%.


242. Which of the following important rivers of India does not originate from the Western Ghats?
A. Cauvery
B. Godavari
C. Krishna
D. Mahanadi

Answer

Answer: Option D
Explanation : The Western Ghats form one of the four watersheds of India, feeding the perennial rivers of India. The major river systems originating in the Western Ghats are Godavari, Kaveri, Krishna, Thamiraparani and Tungabhadra. Mahanadi does not originate from the Western Ghats.


243. Which of the following areas or regions is most prone to earthquakes?
A. Ganga-Brahmaputra valley
B. Deccan plateau
C. Plains of northern India
D. Western ghats

Answer

Answer: Option A
Explanation : Ganga-Brahmaputra valley areas or regions is most prone to earthquakes. Generally, the areas having trap rock or basaltic rock are prone to earthquakes.


244. The proportion of forest to the total national geographical area of India as envisaged by National Forest Policy is
A. 30.3%
B. 33.3%
C. 38.3%
D. 42.3%

Answer

Answer: Option B
Explanation : 33.3% of the total geographical area should be under forest/tree cover.


245. Which of the following dams has generations of power more than irrigation as its main purpose?
A. Gandhi Sagar
B. Hirakud
C. Periyar
D. Tungabhadra

Answer

Answer: Option A
Explanation : Gandhi Sagar dams has generations of power more than irrigation as its main purpose. The Gandhi Sagar Dam is one of the four major dams built on India’s Chambal River. The dam is located in the Mandsaur, Neemuch districts of the state of Madhya Pradesh. It is a masonry gravity dam, standing 62.17 metres (204.0 ft) high, with a gross storage capacity of 7.322 billion cubic metres from a catchment area of 22,584 km2 (8,720 sq mi).


246. Which of the following crops is regarded as a plantation crop?
A. Coconut
B. Cotton
C. Sugarcane
D. Rice

Answer

Answer: Option A
Explanation : Coconut crop is regarded as a plantation crop. The coconut tree (Cocos nucifera) is a member of the palm tree family (Arecaceae) and the only living species of the genus Cocos. The term “coconut” (or the archaic “cocoanut”) can refer to the whole coconut palm, the seed, or the fruit, which botanically is a drupe, not a nut.


247. Which of the following countries leads in the production of aluminium and its products in the world?
A. Australia
B. France
C. India
D. USA

Answer

Answer: Option D
Explanation : Among the following USA leads in the production of aluminium and its products in the world. As a whole, China has the maximum production.


248. The natural region which holds the Indian subcontinent is
A. equatorial climate change region
B. hot deset
C. monsoon
D. mediterranean

Answer

Answer: Option C
Explanation : Monsoon is traditionally defined as a seasonal reversing wind accompanied by corresponding changes in precipitation, but is now used to describe seasonal changes in atmospheric circulation and precipitation associated with the asymmetric heating of land and sea. Usually, the term monsoon is used to refer to the rainy phase of a seasonally changing pattern, although technically there is also a dry phase. The term is sometimes incorrectly used for locally heavy but short-term rains, although these rains meet the dictionary definition of monsoon.


249. The most ideal region for the cultivation of cotton in India is
A. the Brahmaputra valley
B. the Indo-Gangetic valley
C. the Deccan plateau
D. the Rann of Kutch

Answer

Answer: Option C
Explanation : Deccan plateau is the most ideal region for the cultivation of cotton in india. Because of the thick dark soil also known as regur soil which is widely found in Deccan plateau, this place is suitable for cultivation of cotton.


250. Which of the following are true with respect to the Indian Peninsular Plateau?,

A. I II and III
B. I and II
C. I II III and IV
D. I III and IV

Answer

Answer: Option C
Explanation : The following that are true with respect to the Indian Peninsular Plateau are The southern plateau block is formed mainly of granite and gneiss, The Deccan lava plateau is an elevated tableland consisting of horizontally arranged lava sheets, The Malwa plateau dominates the Vindhyam scraps, forming the northern flank of the plateau and The trough of the Narmada and Tapti are interposed between the Vindhyan and the Satpura ranges.


251. Which of the following types of soil are mostly confined to river basins and coastal plains of India?
A. Alluvial soils
B. Black soils
C. Laterite soils
D. Red soils

Answer

Answer: Option A
Explanation : Alluvial soil are mostly confined to river basin and costal plain as river tends to erode the soils and deposit the heavy fertile soil near river and it’s adjacent areas.


252. The two states of India, most richly endowed with iron ore, are
A. Bihar and Orissa
B. Madhya Pradesh and Orissa
C. Bihar and West Bengal
D. Madhya Pradesh and West Bengal

Answer

Answer: Option A
Explanation : The two states of India, most richly endowed with iron ore, are Bihar and Orissa.


253. The most fertile region of India is
A. the Himalayas
B. the central Highlands
C. the Indo-Gangetic plain
D. peninsular plateau

Answer

Answer: Option C
Explanation : The Indo-Gangetic Plain, also known as the Indus-Ganga Plain and the North Indian River Plain, is a 630-million-acre (2.5-million km2) fertile plain encompassing Northern regions of the Indian subcontinent, including most of northern and eastern India, the eastern parts of Pakistan, virtually all of Bangladesh and southern plains of Nepal.


254. Which of the following groups accounts for over 90 per cent of India’s annual coal production?
A. Bihar Orissa and West Bengal
B. Bihar Orissa and Madhya Pradesh
C. Orissa Madhya Pradesh and Tamil Nadu
D. West Bengal Madhya Pradesh and Tamil Nadu

Answer

Answer: Option A
Explanation : Bihar, Orissa and West Bengal accounts for over 90% of India’s annual coal production.


255. The significant shifts in Indian agriculture during green revolution include
A. non-food grains reported some spectacular progress and shift in the cropping pattern
B. major food grains in the eastern regions productions decreased and in the northern states increased
C. both (a) and (b)
D. None of the above

Answer

Answer: Option C
Explanation : The significant shifts in Indian agriculture during green revolution include both non-food grains reported some spectacular progress and shift in the cropping pattern and major food grains, in the eastern regions productions decreased and in the northern states increased.


256. The number of major ports in India is
A. 5
B. 8
C. 13
D. 15

Answer

Answer: Option C
Explanation : There are total 13 major sea ports of India,out of which 12 are government and one, Ennore port of Chennai is the corporate one.


257. Which of the following is a peninsular river of India?
A. Gandak
B. Kosi
C. Krishna
D. Sutlej

Answer

Answer: Option C
Explanation : The peninsular Rivers in India include the Mahanadi, Godavari, Krishna, Cauvery, Narmada, and Tapti or Tapi. Together they drain a significant portion of rural India. These rivers carry both religious and cultural significances in the lives of Indian people. Krishna among the following is a peninsular river of India.


258. Which of the following is the most important raw material for generation of power in India?
A. Coal
B. Mineral Oil
C. Natural Gas
D. Uranium

Answer

Answer: Option A
Explanation : Coal is one of the most important sources of energy and is being used for various proposes such as heating of housed, as fuel for boilers and steam engines and for generation of electricity by thermal plants. Coal is found in abundance in our country and it is the most important source of energy in our country.


259. When it is noon IST at Allahabad in India, the time at Greenwich, London, will be
A. midnight GMT
B. 17 30 hours
C. 06 30 hours
D. None of the above

Answer

Answer: Option C
Explanation : When it is 12:00 pm in Allahabad, there will 6 30 hrs in London as it is 5 hrs 30 minutes behind IST.


260. Which country has the largest coast line?
A. USA
B. Australia
C. Canada
D. India

Answer

Answer: Option C
Explanation : Canada (202,080 km) has the longest total coastlines in the world, and these hold great cultural, military, and economic importance for each of them.


261. The river Godavari is often referred to as Vridha Ganga because
A. it is the older river of India
B. of its large size and extent among the peninsular rivers
C. there are a fairly large number of pilgrimage centres situated on its banks
D. its length is nearly the same as that of the river Ganges

Answer

Answer: Option B
Explanation : The river Godavari is often referred to as Vridha Ganga because of its large size and extent among the peninsular rivers.


262. The scarcity or crop failure of which of the following can cause a serious edible oil crisis in India?
A. coconut
B. Groundnut
C. Linseed
D. Mustard

Answer

Answer: Option B
Explanation : The scarcity or crop failure of Groundnut can cause a serious edible oil crisis in India. The peanut, also known as the groundnut, goober, or monkey nut, and taxonomically classified as Arachis hypogaea, is a legume crop grown mainly for its edible seeds. It is widely grown in the tropics and subtropics, being important to both small and large commercial producers.


263. The pennines (Europe), Appalachians (America) and the Aravallis (India) are examples of
A. old mountains
B. young mountains
C. fold mountains
D. block mountains

Answer

Answer: Option A
Explanation : The pennines (Europe), Appalachians (America) and the Aravallis (India) are examples of old mountains.


264. Which of the following factors are responsible for present crisis in the jute industry in India?,
,
A. I and II
B. I II and III
C. I and III
D. II and III

Answer

Answer: Option A
Explanation : Factors that are responsible for present crisis in the jute industry in India are The decline in overseas market and Inadequately supply of raw jute.


265. Which of the following factors are responsible for India’s failure to fully exploit the inland fisheries during the last five decades?,
,
A. I II and III
B. I and II
C. I and III
D. II and III

Answer

Answer: Option C
Explanation : Factors that are responsible for India’s failure to fully exploit the inland fisheries during the last five decades are Silting and pollution of the inland water bodies and Lack of marketing facilities.


266. Gold is mined in:
A. Karnatka
B. Maharastra
C. Orissa
D. Uttar Pradesh

Answer

Answer: Option A
Explanation : The main minerals found in Karnataka are listed below: Gold: The Kolar Gold Fields in Kolar used to be one of the major producers of gold in India, before it shut down. Presently the gold mines of Hutti and Raichur produce almost 84 per cent of the country’s gold.


267. The source of Kaveri River lies in –
A. Sahyadri
B. Brahmagiri hills
C. Gavaligarh
D. Amarkantak

Answer

Answer: Option B
Explanation : River Kaveri originates in the Brahmagiri hills in Kodagu, in a place called Talakaveri (head of Kaveri). It starts its journey from the small pond called as Kundike pond, later the two tributaries known as Kanake and Sujyoti joins it. All these three rivers meets at the point called Bhagamandala.


268. Which State has the largest reserves of Lignite Coal?
A. Gujarat
B. Kerala
C. Rajasthan
D. Tamil Nadu

Answer

Answer: Option D
Explanation : The largest lignite reserve in India is located at Neyveli in Tamil Nadu. At places, these coal seams are more than 15 metres thick. This coal has more than 35 per cent carbon content. Neyveli Lignite Corporation has been set up for multipurpose use of this coal, after enrichment of its carbon content.


269. Which of the following rivers makes an estuary?
A. Krishna
B. Kaveri
C. Ganga
D. Narmada

Answer

Answer: Option D
Explanation : The Narmada, Periyar and Tapti are the only long rivers, which flow west and make estuaries. An Estuary is a partly enclosed coastal body of water with one or more rivers or streams flowing into it, and with a free connection to the open sea. Only West following rivers of India forms Estuary.


270. Which of the following is known as the morning star?
A. Saturn
B. Jupiter
C. Mars
D. Venus

Answer

Answer: Option D
Explanation : Because it seems In addition to being known as the evening star, Venus was also called the morning star because it could be seen for a few hours before the Sun grew too bright. The planet actually becomes brightest before the Sun rises or just after sunset.


271. Which of the following is largest planet?

A. Jupiter
B. Neptune
C. Saturn
D. Mars

Answer

Answer: Option A
Explanation : The largest planet in our solar system by far is Jupiter, which beats out all the other planets in both mass and volume. Jupiter’s mass is more than 300 times that of Earth, and its diameter, at 140,000 km, is about 11 times Earth’s diameter.


272. Black soil is favourable for cultivation of-
A. Cotton
B. Coffee
C. Rice
D. Tea

Answer

Answer: Option A
Explanation : Black soils are poor in nitrogen, phosphorus and organic matter. The soils are generally rich in the montmorillonitic and beidellitic group of clay minerals. It is most suitable for the growth of cotton. It is also known or cotton soil as a reason.


273. The largest producer of Sugar in India is

A. Bihar
B. Karnataka
C. Maharashtra
D. Uttar Pradesh

Answer

Answer: Option D
Explanation : Uttar Pradesh (UP) will be the largest producer of sugar in India; followed by Maharashtra, which is expected to trail marginally behind UP. Also, sugar production in Karnataka will recover to its near normal levels (five-year average).


274. The Sahyadri is another name of:
A. Western Ghat
B. Vindhya Range
C. Shivalik Range
D. Eastern Ghat

Answer

Answer: Option A
Explanation : Western Ghats also known as Sahyadri (Benevolent Mountains) is a mountain range that covers an area of 140,000 km² in a stretch of 1,600 km parallel to the western coast of the Indian peninsula, traverse the States of Kerala, Tamil Nadu, Karnataka, Goa, Maharashtra and Gujarat.


275. Which of the following pairs is correct?
A. Maharashtra – Paradeep
B. Tamil Nadu – Mangalore
C. Gujarat – Kandla
D. Karnataka – Tuticorin

Answer

Answer: Option C
Explanation : Kandla is a seaport in Kutch district of Gujarat in western India.


Geography MCQ Quiz Questions
MCQ Questions on Indian Geography MCQ Questions on Universe
MCQ Questions on Earth MCQ Questions on Atmosphere
MCQ Questions on Hydrosphere MCQ Questions on Lithosphere
MCQ Questions on Political Geography of India MCQ Questions on Climate and Weather in Indian Geography
MCQ Questions on Agriculture and Soil in Indian Geography MCQ Questions on Indian Rivers, Lake, and Water
MCQ Questions on Natural Resources and Industries in Indian Geography MCQ Questions on Transportation system of India
MCQ Questions on Environment and Ecology in Indian Geography MCQ Questions on Population and Tribes in Indian Geography
MCQ Questions on Miscellaneous Indian Geography MCQ Questions on World Climate and Weather
MCQ Questions on World Agriculture, Minerals and Industries MCQ Questions on World Environment and Ecology
MCQ Questions on Miscellaneous World Geography

Leave a Comment